Sie sind auf Seite 1von 120

GMATCLUBFLASHCARDS

ContributingtoEachOthersLearning
http://gmatclub.com/flashcards
GMAT Club 2011 1
GMAT Club contributing to each others learning
GMAT Club Marketplace
Find this and other great free products are available
for download from the GMAT Club Marketplace.
Find the best free resources and also the best
deals to save the most money on books, tests,
courses, iPhone Apps, and admissions consulting.

http://gmatclub.com/marketplace

GMAT Club 2011 2
GMA T MA TH
GMAT Club 2011 3
GMAT Club contributing to each others learning
Percent
What is the percent formula?


How much is 15 percent of 20?
MATH: ARITHMETIC
1
GMAT Club contributing to each others learning
Percent | Answers
Percent Formula:
Part = Percent x Whole

For example, 15% of 20 is:
15
100
20 =
300
100
= 3
MATH: ARITHMETIC
2
GMAT Club contributing to each others learning
Percent 2
What is the formula for percent change?

If Jack got a raise from $15 per hour to $18
per hour, what was the percent increase?
MATH: ARITHMETIC
3
GMAT Club contributing to each others learning
Percent 2 | Answers
Percent Change Formula:


100%

Jacks Raise:
18 15
15
100% =
3
15
100% =
100
5
= 20%
! Make sure you use the original amount (15), not
the new one (18)
MATH: ARITHMETIC
4
GMAT Club 2011 4
GMAT Club contributing to each others learning
Percent 3
If the production of hybrid cars tripled last year, by
how many percent did it increase?
100%
200%
250%
300%
333%
MATH: ARITHMETIC
5
GMAT Club contributing to each others learning
Percent 3 | Answers
The correct answer is B increased by 200%
For example, if production was 10 cars, and it
tripled to 30 cars, the increase was 20 cars, which
is 200% of 10
MATH: ARITHMETIC
6
GMAT Club contributing to each others learning
Percent 4


50% of 25 is 25% of which number?
MATH: ARITHMETIC
7
GMAT Club contributing to each others learning
Percent 4 | Answers
There are 2 solutions to this problem:
Long one:
50% of 25 is 12.5
12.5 *100 = x * 25
x = 50
Short one but a harder to come up with:
50% * 25 = x * 25%
50 = x
MATH: ARITHMETIC
8
GMAT Club 2011 5
GMAT Club contributing to each others learning
Odd/Even Rules
Check your knowledge of Odd/Even rules
Odd + Odd = ?
Odd Even = ?
Even + Odd = ?
Odd x Odd = ?
Odd Even = ?
Odd x Even = ?
Hint: try picking numbers
MATH: ARITHMETIC
9
GMAT Club contributing to each others learning
Odd/Even Rules | Answers
Odd + Odd = 1+ 1 = 2 (Even)
Odd Even = 3 2 = 1 (Odd)
Even + Odd = 2 + 1 = 3 (Odd)
Odd x Odd = 3 x 3 = 9 (Odd)
Odd Even = 3 2 = 1.5 (Not an integer!)
Odd x Even = 3 x 2 = 6 (Even)
MATH: ARITHMETIC
10
GMAT Club contributing to each others learning
Odd/Even Rules 2


Is 0 odd or even?
MATH: ARITHMETIC
11
GMAT Club contributing to each others learning
Odd/Even Rules 2 | Answers


0 (zero) is Even
It is also not positive or negative
it is neutral
MATH: ARITHMETIC
12
GMAT Club 2011 6
GMAT Club contributing to each others learning
Odd/Even Rules 3
If b is an odd number, which of the following
must be even?
A. 2 3
B.
;1
2

C.

2
1
D.
2
+2
E. 2 +2
MATH: ARITHMETIC
13
GMAT Club contributing to each others learning
Odd/Even Rules 3 | Answers
Use the plug numbers method to check each
answer choice. Lets take 1:
A. 2*1 3 = -1 (Odd)
B.
1;1
2
= 0 (Even) however, just in case, lets try 3.
3;1
2
= 1 (Odd). It usually is a good idea to run
through 2 different numbers if you get zero or similar
C.
1
2
1 =
1
2
(not an intenger)
D. 1 + 2 = 3 (Odd)
E. 2 + 2 = 4 (Even)
MATH: ARITHMETIC
14
GMAT Club contributing to each others learning
Odd/Even Rules 4 (Ultra Hard)
Is A+B+C even or odd?
A C B is even

;

is odd
MATH: ARITHMETIC
15
GMAT Club contributing to each others learning
Odd/Even Rules 4 | Answers
Statements (1) and (2) combined are insufficient.
Consider A=6, B=4, and C=2 (the answer is "yes")
and A=0.5, B=0.3, and C=0.2 (the answer is "no").
Do not assume that the numbers are integers if
the question does not mention it.
The correct answer is E.

MATH: ARITHMETIC
16
GMAT Club 2011 7
GMAT Club contributing to each others learning
Divisibility Rules
Is 54780 divisible by 2?
Is 1671 divisible by 3?
Is 5632 divisible by 4?
Is 3830 divisible by 5?
Is 2658 divisible by 6?
Is 396 divisible by 9?

MATH: ARITHMETIC
17
GMAT Club contributing to each others learning
Divisibility Rules | Answers
Yes. Divisibility by 2 last digit of a number is even
Yes. Divisibility by 3 sum of all digits is a multiple
of 3
Yes. Divisibility by 4 last 2 digits is a multiple of 4
Yes. Divisibility by 5 the last digit is either 0 or 5
Yes. Divisibility by 6 the sum of digits is a multiple
of 3 and the last digit is even
Yes. Divisibility by 9 the sum of digits is a multiple
of 9

MATH: ARITHMETIC
18
GMAT Club contributing to each others learning
Divisibility 2 (Hard)
Is integer
2

4
divisible by 9?
x is an integer divisible by 3
xy is an integer divisible by 9
MATH: ARITHMETIC
19
GMAT Club contributing to each others learning
Divisibility 2 | Answers
The best way to approach this question is to plug
in several sets of numbers
Many are tempted to plug 3 for x and then for S2,
the only value y can have is 3; in that case, the
answer is Yes.
But if we try x=81 and y=
1
9
, then x is an integer
divisible by 3, xy is an integer divisible by 9, but

4
= 1 and is not divisible by 9.
The answer is E
MATH: ARITHMETIC
20
GMAT Club 2011 8
GMAT Club contributing to each others learning
Number Properties
Which of the following integers represents a sum
of 3 consecutive even integers?
200
303
400
554
570
MATH: ARITHMETIC
21
GMAT Club contributing to each others learning
Number Properties | Answers
To answer the question, check which integer is
both divisible by 3 (since there are 3 integers) and
is even. The only number that falls into both of
those 2 categories is 570.
Correct answer is E
MATH: ARITHMETIC
22
GMAT Club contributing to each others learning
Number Properties 2 (Hard)
If
2
= + 5, = 2, =
2,
3
+
2
+ 7?
x > 0
y = 4
MATH: ARITHMETIC
23
GMAT Club contributing to each others learning
Number Properties 2 | Answers
Substitute x, y, and z in the original equation to get the
following:
2
2 3 = 0; The solutions to this
equation are x=-1 and x=3.
Statement 1 is sufficient. If x>0, then the only solution
is x=3. The result is 27+16+6 =49, which is divisible by
7
Statement 2 is sufficient. If y=4, plugging in this value
into
2
= +5, gives us that x=3 or x=-3. However,
based on the above, x=3 or x=-1. Therefore x=3.
Sufficient.
The correct answer is D.
MATH: ARITHMETIC
24
GMAT Club 2011 9
GMAT Club contributing to each others learning
Number Properties 3
How many distinct integers are
there between 1 and 21 inclusive?
MATH: ARITHMETIC
25
GMAT Club contributing to each others learning
Number Properties 3 | Answers
Most often GMAT questions will say inclusive but
sometimes they dont need to remember to
check
The formula for calculating the number of integers
between two numbers is: N-M+1
Therefore, the answer is: 21 1 + 1 = 21
You can also write out all of the numbers though it
is not always possible but just in case:
1, 2, 3, 4, 5, 6, 7, 8, 9, 10, 11, 12, 13, 14, 15, 16, 17,
18, 19, 20, 21
MATH: ARITHMETIC
26
GMAT Club contributing to each others learning
Multiples
What is a multiple?

Is 5 a multiple of 55 or is 55 a
multiple of 5?
MATH: ARITHMETIC
27
GMAT Club contributing to each others learning
Multiples | Answers
The multiple of a number is the
product of the number and any
other whole number. (For example,
2,4,6,8 are multiples of 2)
Therefore 55 is a multiple of 5
For example, 6 has factors 1, 2, 3, 6;
and multiples that are 6, 12, 24, 36
MATH: ARITHMETIC
28
GMAT Club 2011 10
GMAT Club contributing to each others learning
Multiples 2


Is 0 (zero) a multiple of 100?
MATH: ARITHMETIC
29
GMAT Club contributing to each others learning
Multiples 2 | Answers

Yes, 0 (zero) is a multiple of
everything
However, it is unlikely that GMAT
will test this property but it helps to
remember
MATH: ARITHMETIC
30
GMAT Club contributing to each others learning
Multiples 3



How to find a Least Common
Multiple (LCM)?

MATH: ARITHMETIC
31
GMAT Club contributing to each others learning
Multiples 3 | Answers
Option 1:
Step 1: Find the prime factors of each of the numbers
Step 2: Multiply the unique factors (exclude
duplicates)
Option 2:
Step 1: Multiply the two numbers
Step 2: Find any factors the two numbers share
Step 3: Divide the product in Step 1 by the factors
that the two numbers have in common
MATH: ARITHMETIC
32
GMAT Club 2011 11
GMAT Club contributing to each others learning
Multiples 4


What is the Least Common
Multiple of 18 and 24?
MATH: ARITHMETIC
33
GMAT Club contributing to each others learning
Multiples 2 Answers
Option 1
Find the factors of 18 and 24:
18 = 3 x 3 x 2
24 = 3 x 2 x 2 x 2
Multiply the unique factors: 3 x 3 x 2 x 2 x 2 = 72
Option 2
Multiply the 2 numbers: 18 x 24 = 432
Shared factors: 2 and 3
Divide 432 by 2 and 3 = 72
MATH: ARITHMETIC
34
GMAT Club contributing to each others learning
Translate

The product of three and four is reduced by
five and then increased by the difference
between the original product and eight = ?
MATH: ARITHMETIC
35
GMAT Club contributing to each others learning
Translate Answers

3 x 4 = 12
12 5 = 7
12 8 = 4
7 + 4 = 11
Answer: 11
MATH: ARITHMETIC
36
GMAT Club 2011 12
GMAT Club contributing to each others learning
Arithmetic to Memorize

1
2
=

1
4
=

2
5
=

1
20
=

1
8
=

1
6
=
MATH: ARITHMETIC
37
GMAT Club contributing to each others learning
Arithmetic to Memorize | Answers

1
2
= 50%

1
4
= 25%

2
5
= 40%

1
20
= 5%

1
8
= 12.5%

1
6
= 16.67%

MATH: ARITHMETIC
38
GMAT Club contributing to each others learning
Arithmetic to Memorize 2

1
12
=

7
8
=

3
4
=
75% =
20% =
16
2
3
% =
83
1
3
% =
MATH: ARITHMETIC
39
GMAT Club contributing to each others learning
Arithmetic to Memorize 2 | Answers

1
12
= 8.33%

7
8
= 87.5%

3
4
= 75%
75% =
20% =
1
5

16
2
3
% =
1
6

83
1
3
% =
5
6


MATH: ARITHMETIC
40
GMAT Club 2011 13
GMAT Club contributing to each others learning
Arithmetic to Memorize 3
2
2
=
2
3
=
2
4
=
2
5
=
2
6
=
2
7
=
MATH: ARITHMETIC
41
GMAT Club contributing to each others learning
Arithmetic to Memorize 3 | Answers
2
2
= 4
2
3
= 8
2
4
= 16
2
5
= 32
2
6
= 64
2
7
= 128

MATH: ARITHMETIC
42
GMAT Club contributing to each others learning
Arithmetic to Memorize 4
3
2
=
3
3
=
3
4
=
3
5
=
4
2
=
4
3
=
4
4
=

MATH: ARITHMETIC
43
GMAT Club contributing to each others learning
Arithmetic to Memorize 4 | Answers
3
2
= 9
3
3
= 27
3
4
= 81
3
5
= 243
4
2
= 16
4
3
= 64
4
4
= 256

MATH: ARITHMETIC
44
GMAT Club 2011 14
GMAT Club contributing to each others learning
Arithmetic to Memorize 5
5
2
=
5
3
=
11
2
=
12
2
=
13
2
=
14
2
=
15
2
=
16
2
=
MATH: ARITHMETIC
45
GMAT Club contributing to each others learning
Arithmetic to Memorize 5 | Answers
5
2
= 25
5
3
= 125
11
2
= 121
12
2
= 144
13
2
= 169
14
2
= 196
15
2
= 225
16
2
= 256
MATH: ARITHMETIC
46
GMAT Club contributing to each others learning
Arithmetic to Memorize 6

Complete the following:
8, 16, 24, 32, 40, 160

12, 24, 36. 120

15, 30 150

MATH: ARITHMETIC
47
GMAT Club contributing to each others learning
Arithmetic to Memorize 6 | Answers

Complete the following (Answers):
8, 16, 24, 32, 40, 48, 56, 64, 72, 80, 88, 96, 104,
112, 120

12, 24, 36, 48, 60, 72, 84, 96, 108, 120

15, 30, 45, 60, 75, 90, 105, 120

MATH: ARITHMETIC
48
GMAT Club 2011 15
GMAT Club contributing to each others learning
Arithmetic to Memorize 7
2 =
3 =
625 =
169 =
List all Primes between 1 and 50
Extra Hard & Extra Credit
2
6
=
125
3
=
243
5
=
MATH: ARITHMETIC
49
GMAT Club contributing to each others learning
Arithmetic to Memorize 7 | Answers
2 = 1.4
3 = 1.7
625 = 25
169 = 13
Primes: 2, 3, 5, 7, 11, 13, 17, 19, 23, 29, 31, 37, 41, 43,
47
2
6
= 64
125
3
= 5
243
5
= 3


MATH: ARITHMETIC
50
GMAT Club contributing to each others learning
Reciprocal


What is a reciprocal?
MATH: ARITHMETIC
51
GMAT Club contributing to each others learning
Reciprocal Answers
Reciprocal for a number , denoted by
1

or

;1
, is a number which when multiplied by
yields 1. The reciprocal of a fraction

is

.
For example reciprocal of 3 is
1
3

Reciprocal of
5
6
is
6
5
.
MATH: ARITHMETIC
52
GMAT Club 2011 16
GMAT Club contributing to each others learning
Absolute Value


What is the 3- step approach to solving
equations and inequalities with absolute
value?
MATH: ARITHMETIC
53
GMAT Club contributing to each others learning
Absolute Value | Answers
Step1: Open modulus and set conditions.
To solve/open a modulus, you need to
consider 2 situations to find all roots:
Positive (or rather non-negative)
Negative
Step 2: Solve new equations from Step 1
Step 3: Check conditions for each
solution from Step 2
MATH: ARITHMETIC
54
GMAT Club contributing to each others learning
Absolute Value 2

If |x-1| = 4, what is the value of
x?
MATH: ARITHMETIC
55
GMAT Club contributing to each others learning
Absolute Value 2 | Answers
Step 1: Find positive/negative roots:
1 0 in our case, 1 = 4
1 < 0 in our case, 1 = 4
Step 2: Solve the equations
x = 5
x = -3
Step 3: Check conditions ( 1 0 & 1 < 0)
5 1 = 4 0
-3 1 = -4 < 0

MATH: ARITHMETIC
56
GMAT Club 2011 17
GMAT Club contributing to each others learning
Absolute Value 3


What is the value of x If |x 5| = 10?
MATH: ARITHMETIC
57
GMAT Club contributing to each others learning
Absolute Value 3 | Answers
|x 5| = 10
We must evaluate both the positive and negative
outcome since the absolute value removes the
negative sign
Thus, we have 2 equations:
X 5 = 10
X 5 = - 10 (in case it was a negative expression)
Thus x = 15 and -5. (Plug in both to check)
Absolute value inequalities and equations will almost
always have 2 answers/solutions!

MATH: ARITHMETIC
58
GMAT Club contributing to each others learning
Absolute Value 4
Is M<0?
= ||

2
= 9
MATH: ARITHMETIC
59
GMAT Club contributing to each others learning
Absolute Value 4 | Answers
Statement (1) by itself is not sufficient. From
statement (1), M is either a negative number or
zero. If M=-3, then (-3)=|-3| or 3=3, which is not
sufficient.
Statement (2) by itself is not sufficient. From
statement (2), M can be either 3 or -3, which is not
sufficient.
Statements (1) and (2) combined are sufficient. If
we combine both statements, then M=-3.
The correct answer is C.
MATH: ARITHMETIC
60
GMAT Club 2011 18
GMAT Club contributing to each others learning
Factors


How many total factors does 462
have?
MATH: ARITHMETIC
61
GMAT Club contributing to each others learning
Factors | Answers
Advanced method for finding the number of all possible
factors:
Write out each prime factor and their power, for
example, 12 can be written as follows:
2
2

3
1

5
0
etc (all other primes will have a zero power)
Add 1 to all powers and multiply them: (in this case 6)
and thats the number of factors (1, 2, 3, 4, 6, 12)
Prime Factors of 462 = 2
1
, 3
1
, 7
1
, 11
1

Total number of factors: 2x2x2x2 = 2
4
= 16
MATH: ARITHMETIC
62
GMAT Club contributing to each others learning
Factors 2


Do integers usually have an odd or
even number of factors?
MATH: ARITHMETIC
63
GMAT Club contributing to each others learning
Factors 2 | Answers
Vast majority of integers have an even number of
factors such as 12 for example: 1, 2, 3, 4, 6, 12.
Thus when finding the total number of factors,
make sure it is an even number
The only integers that have an odd number of
factors are perfect squares (thats numbers such as
4, 9, 16, 25, 36, etc). Try 25 for example, it has
factors of 1, 5, and 25. Factors of 36 are 1, 2, 3, 4,
6, 9, 12, 18, 36.
MATH: ARITHMETIC
64
GMAT Club 2011 19
GMAT Club contributing to each others learning
Ratios
What is the ratio of a to d if
2a=3b,
1
2
=2c, and 3c=d?

A)1:2 B)2:1 C)2:3 D)4:3 E)1:5

MATH: ARITHMETIC
65
GMAT Club contributing to each others learning
Ratios
With questions like these, unless you can spot a
shortcut right away, the easiest way to solve is to
plug numbers: lets pick a=6, since there is a 2 and
3 involved
2*6 = 3b; b = 4
0.5*4 = 2c; c = 1
3*1 = d; d = 3
Therefore, the ration of a to d is 2:1
The correct answer is B.
MATH: ARITHMETIC
66
GMAT Club contributing to each others learning
Powers

2
2
+2
3
+ 2
4
+ 2 = ?
A) 2
9
+ 2
B) 2
10

C) 2
5
+ 2
D) 2
5

E) 30
MATH: ARITHMETIC
67
GMAT Club contributing to each others learning
Powers | Answers


2
2
+ 2
3
+2
4
+2 = ?
There are no rules for adding or
subtracting powers. This question is
solved by brute force:
4 + 8 + 16 + 2 = 30
The correct answer is (E)
MATH: ARITHMETIC
68
GMAT Club 2011 20
GMAT Club contributing to each others learning
Powers 2
2
2
2
3
2
4
2
5
2 = ?
A) 2
120

B) 32
15

C) 2
15

D) 2
14

E) 62
MATH: ARITHMETIC
69
GMAT Club contributing to each others learning
Powers 2 | Answers
2
2
2
3
2
4
2
5
2 = 2
15

When powers with the same base (2
in this case) are multiplied, the powers
are summed and the base is held
constant. Thus, it is 2 to the power of
2+3+4+5+1=15.
The correct answer is (C) 2
15

MATH: ARITHMETIC
70
GMAT Club contributing to each others learning
Powers 3


2
4
2
8
2
6
2
=?
MATH: ARITHMETIC
71
GMAT Club contributing to each others learning
Powers 3 | Answers


2
4
2
8
2
6
2
=
2
12
2
7
= 2
5

Division of powers with the same base
(2) is handled similarly to multiplication
except the power values are deducted.
The base stays unchanged

MATH: ARITHMETIC
72
GMAT Club 2011 21
GMAT Club contributing to each others learning
Powers 4

(3
3
)
2
= ?
MATH: ARITHMETIC
73
GMAT Club contributing to each others learning
Powers 4 | Answers

(3
3
)
2
= 3
6

When a number taken to a power is
taken to another power, you need to
multiply the exponents (2 * 3)
MATH: ARITHMETIC
74
GMAT Club contributing to each others learning
Powers 5


3
3
3
= ?


MATH: ARITHMETIC
75
GMAT Club contributing to each others learning
Powers 5 | Answers


3
3
3
= 3
27

Start operations from outside and work your way
in: 3 taken to the power of 3 is 27. Thus the base of
3 is taken to the power of 27.

MATH: ARITHMETIC
76
GMAT Club 2011 22
GMAT Club contributing to each others learning
Powers 6


2
;2
2
;3
2
6
2
;4
=?
MATH: ARITHMETIC
77
GMAT Club contributing to each others learning
Powers 6 | Answers

2
;2
2
;3
2
6
2
;4
= 2
;3
=
1
2
3
=
1
8

Negative powers turn a number to a
reciprocal number taken to that power.
For example 2
;2
=
1
2
2
=
1
4

MATH: ARITHMETIC
78
GMAT Club contributing to each others learning
Powers 7



5
0
= ?
MATH: ARITHMETIC
79
GMAT Club contributing to each others learning
Powers 7 | Answers

5
0
= 1
Any number (positive or
negative) taken to the power of 0
(zero) equal to 1.
Not tested on the GMAT, zero to
the power of zero is also 1.
MATH: ARITHMETIC
80
GMAT Club 2011 23
GMAT Club contributing to each others learning
Powers 8


4
1
2
= ?
MATH: ARITHMETIC
81
GMAT Club contributing to each others learning
Powers 8 | Answers


4
1
2
= 4 = 2
Fraction powers are interpreted as
follows: the denominator is the root and
numerator is the power. For example
3
2
3
= 3
3
2
or 2
1
2
= 2
MATH: ARITHMETIC
82
GMAT Club contributing to each others learning
Powers 9 (Ultra Hard)
If m and n are positive integers, is the remainder of
10

:
3
larger than the remainder of
10

:
3
?
m > n
The remainder of

3
is 2
MATH: ARITHMETIC
83
GMAT Club contributing to each others learning
Powers 9 | Answers
Statement (1) by itself is insufficient. In expression
10

+, the sum of digits of 10

is always 1 and it is
the value of n that determines the remainder of
10

:
3

. If you plug in m=2, n=1 (the answer is "yes") and m=3,
n=2 (the answer is "no").
Statement (2) by itself is sufficient. If the remainder of

3
is 2, as S2 states, then is 2, 5, or 8 and the sum of the
digits of
10

:
3
is divisible by 3. Therefore, the
remainder of
10

:
3
is 0, which cannot be larger that
the remainder of
10

:
3
no matter what m is.
The correct answer is B.

MATH: ARITHMETIC
84
GMAT Club 2011 24
GMATClub contributingtoeachotherslearning
Percentiles
Lenasfirsttestscorewasatthe80
th
percentilein
aclassof120students.Onanothertest,24outof
200studentsscoredbetterthanLena.Ifnobody
hadLenasscore,whatisLenaspercentileafter
thetwotests?
MATH:ARITHMETIC
85
GMATClub contributingtoeachotherslearning
Percentiles|Answers
Takeeachtestresultseparately:
Test1:120x80
th
=96(shescoredbetterthan96
students)
Test2:Shescoredbetterthan176students(200 24)
Sumuptheresults:
96+176=272
120+200=320

272
320
= 8S
th
percentile
MATH:ARITHMETIC
86
GMATClub contributingtoeachotherslearning
ConsecutiveNumbers
Whatisthesumofconsecutiveintegers
9,8,7,6,5,4,3,2,1,0,1,2?
MATH:ARITHMETIC
87
GMATClub contributingtoeachotherslearning
ConsecutiveNumbers|Answers
Sumofconsecutiveintegersequalsthemean
multipliedbythenumberofterms,n.Given
consecutiveintegers9,8,7,6,5,4,3,2,1,0,1,2
mcon =
-9+2
2
= -S.S (meanequalstothe
averageofthefirstandlastterms),sothesum
equalsto-S.S 12 = -42 .
MATH:ARITHMETIC
88
GMAT Club 2011 25
GMAT Club contributing to each others learning
Evenly Spaced Set



What is the sum of all members of
the set 9,12,15,18,21,24 ?
MATH: ARITHMETIC
89
GMAT Club contributing to each others learning
Evenly Spaced Set | Answers
The sum of elements of evenly spaced set is given
by the formula
Sum =

1
:

2

Therefore,
9:24
2
6 = 99
MATH: ARITHMETIC
90
GMAT Club contributing to each others learning
Recurring Decimal



Express 0.393939 in a fraction format
MATH: ARITHMETIC
91
GMAT Club contributing to each others learning
Recurring Decimal | Answers
To convert a recurring decimal to fraction:
1. Separate the recurring number from the decimal
fraction
2. Annex denominator with "9" as many times as the
length of the recurring number
3. Reduce the fraction to its lowest terms

Example #1: Convert 0.3939 to a fraction
1: The recurring number is 39
2:
39
99
- the number is 2 digits so two nines are added
3: Reducing it to lowest terms:
13
33

MATH: ARITHMETIC
92
GMAT Club 2011 26
GMAT Club contributing to each others learning
Fractions (Extra Hard)
If a, b, and c are positive distinct
integers, is
(

an integer?
c = 2
a = b + c
MATH: ARITHMETIC
93
GMAT Club contributing to each others learning
Fractions | Answers
Statement I is insufficient since it does not provide
enough information about b or c
Statement II: We can rewrite
(

as


Now plug in the value for a from S2:
:

=
1

+
1


since b and c are disticnt positive integers and b is
not equal to c, the expression cannot be an integer
The correct answer is B

MATH: ARITHMETIC
94
GMAT Club 2011 27
GMATClub contributingtoeachotherslearning
Algebra(Hard)
Ifeachexpressionunderthesquarerootisgreater
thanorequalto0,whatis x
2
- 6x + 9 +
2 - x + x - S?
2 - x
2x -6 + 2 - x
2 - x + x - S
2x -6 + x - 2
x + x - 2
MATH:ALGEBRA
1
GMATClub contributingtoeachotherslearning
Algebra|Answers
Basedonthesetup(andMathprinciplestestedon
theGMAT),(2 - x) hastobe~ u.Therefore
x < 2.
x
2
- 6x + 9 = x - S
2
= S - x
Becausex < 2,x - S < u
Therefore:S - x + 2 - x + x - S = 2 - x
ThecorrectanswerisA
MATH:ALGEBRA
2
GMAT Club 2011 28
GMATClub contributingtoeachotherslearning
Average
Iftheaverageof5consecutiveintegersis
12,whatistheaverageoftheevenonly
integers?
10
12
13.5
18
36
MATH:STATISTICS
1
GMATClub contributingtoeachotherslearning
Average|Answers
First,findtheconsecutiveintegers.Since
thereare5,theremaybeeither2or3even
integers.Theseintegersare10,11,12,13,14.
Theaverageoftheevenintegersis12aswell
(10+12+14=36.Divide36by3andyouwill
get12).
ThecorrectanswerisB.
MATH:STATISTICS
2
GMATClub contributingtoeachotherslearning
Average2
Theaverageof10consecutiveintegersis12.
Then,9isdeductedfromthefirstconsecutive
number,8isdeductedfromthesecond,7is
deductedfromthethird,andsoonuntilthe
lastnumberwhichremainsunchanged.What
isthenewaverage?
A)55 B)50 C)6 D)7.5 E)7
MATH:STATISTICS
3
GMATClub contributingtoeachotherslearning
Average2|Answers
Youdontneedtofindeachofthenumbers.
Instead,youhavetwooptions,youcandeductthe
averageofnumbersbetween0and9(thereisa
trapthough,thereshouldbe10numbersrather
than9andtheaverageis4.5,not5)oryoucan
findthesumofconsecutiveintegers(10*12=120)
andsubtractthesumofintegersfrom0to9(45)
anddividetheresult(75)by10.
ThecorrectanswerisE.7.5
MATH:STATISTICS
4
GMAT Club 2011 29
GMATClub contributingtoeachotherslearning
Mean
HowtofindtheMean?
MATH:STATISTICS
5
GMATClub contributingtoeachotherslearning
Mean
ArithmeticMean=Average=
Sum o clcmcnts
numbei of elements
MATH:STATISTICS
6
GMATClub contributingtoeachotherslearning
Median
HowtofindtheMedian?
MATH:STATISTICS
7
GMATClub contributingtoeachotherslearning
Median|Answers
Arrangeallnumbersinanorder
fromthesmallesttothelargest.The
Median willbethemiddlenumber.
Ifthereisanevennumberof
elements,theMedian willbethe
averageofthe2middlenumbers
MATH:STATISTICS
8
GMAT Club 2011 30
GMATClub contributingtoeachotherslearning
Mode
Howtofindthemode?
MATH:STATISTICS
9
GMATClub contributingtoeachotherslearning
Mode|Answers
TheModeofanarrayisthenumberthat
appearsmostoften.Forexample,inan
array1,2,3,3,4 theMode is3.It
appearedtwice.Inthearray1,2,3,3,4,4
theMode is3and4(therecanbemore
thanonemode).
MATH:STATISTICS
10
GMATClub contributingtoeachotherslearning
Range
HowtofindtheRange?
MATH:STATISTICS
11
GMATClub contributingtoeachotherslearning
Range|Answers
Rangeisthedifferencebetweenthe
smallestandlargestelementsofan
array.Ifyouhavetofindmorethanjust
range,itisalwaysagoodideatorewrite
theelementsinancreasingorder
MATH:STATISTICS
12
GMAT Club 2011 31
GMATClub contributingtoeachotherslearning
StandardDeviation
HowtofindtheStandard
Deviation?
MATH:STATISTICS
13
GMATClub contributingtoeachotherslearning
StandardDeviation|Answers
YouwonthavetocalculateSDontheGMATbut
youneedtounderstandtheconceptofSD
StandardDeviationmeasureshowspreadoutthe
membersofthearrayare.TofindtheStandard
Deviation:
Findthemean
Findthedifferencebetweeneachnumberandthe
mean
Squareeachofthedifferences
Findtheaverageofthesquareddifferences
Takeasquarerootoftheaverage
MATH:STATISTICS
14
GMATClub contributingtoeachotherslearning
StandardDeviation2
Whichofthesetshasahigherstandarddeviation?
SetA SetB

MATH:STATISTICS
15
GMATClub contributingtoeachotherslearning
StandardDeviation2|Answers
SetAhasthehigherStandardDeviationbecause
theelementsaredistributedfurtherfromthe
mean
MATH:STATISTICS
16
GMAT Club 2011 32
GMATClub contributingtoeachotherslearning
StandardDeviation3
Whatisthefastestwaytoestimatestandard
deviation(withoutcalculatingit)?
Thereisaset{67,32,76,35,101,45,24,37}.Ifwe
createanewsetthatconsistsofallelementsof
theinitialsetbutdecreasedby17%,whatisthe
changeinstandarddeviation?
MATH:STATISTICS
17
GMATClub contributingtoeachotherslearning
StandardDeviation3|Answers
Wedon'tneedtocalculateasdecreaseinall
elementsofasetbyaconstantpercentagewill
decreasethestandarddeviationofthesetbythe
samepercentage(theaverageisdecreasedby17%
aswellasthedifferencebetweenaverage(mean)
andallelementsortheirsquares.Thusthe
decreaseinstandarddeviationis17%.
Youcanalwaystrytomodelasetof3numbers
P.S.the17%hasbeenverifiedinExcel
MATH:STATISTICS
18
GMATClub contributingtoeachotherslearning
StandardDeviation4
WhatistheStandardDeviationofaset
ofconsecutiveevenintegers?
(1)Thereare39elementsintheset.
(2)themeanofthesetis382
MATH:STATISTICS
19
GMATClub contributingtoeachotherslearning
StandardDeviation4|Answers
BeforereadingDataSufficiencystatements,whatcan
wesayaboutthequestion?Whatshouldweknowto
findstandarddeviation?"consecutiveevenintegers"
meansthatallelementsstrictlyrelatedtoeachother.
Ifweshiftthesetbyaddingorsubtractinganyinteger,
itdoesnotchangethestandarddeviation.Onething
weshouldknowisthenumberofelementsintheset,
becausethemoreelementswehavethebroaderthey
aredistributedrelativetothemean.Now,lookatDS
statements,allweneeditisjustfirststatement.So,A
issufficient.
MATH:STATISTICS
20
GMAT Club 2011 33
GMATClub contributingtoeachotherslearning
StandardDeviation5
SetAconsistsof19integerswithmean4and
standarddeviationof3.IfanewsetBisformedby
adding2moreelementstothesetA,whattwo
elementswilldecreasethestandarddeviationthe
most?
A)9and3
B)3and3
C)6and1
D)4and5
E)5and5
MATH:STATISTICS
21
GMATClub contributingtoeachotherslearning
StandardDeviation5|Answers
Solution:Theclosertothemean,thesmaller
thestandarddeviation,andtherefore,the
greaterthedecreaseinstandarddeviation.D
has4(equaltothemean)and5(differsfrom
meanonlyby1).
Allotheroptionshaveagreaterdeviation
fromthemean
Answer:D
MATH:STATISTICS
22
GMAT Club 2011 34
GMATClub contributingtoeachotherslearning
CompoundInterest
MATH:WORDPROBLEMS
If$20,000isinvestedat12%annual
interest,compoundedquarterly,whatis
thebalanceafter1year?
1
GMATClub contributingtoeachotherslearning
CompoundInterest|Answers
MATH:WORDPROBLEMS
CompoundInterestformula:
= principol 1 +
Intcrcst
C
ycus c
whereC isthenumberofperiods
2u,uuu 1 +
0.12
4
1 4
= 2u,uuu 1.uS
4
1.03tothe4
th
poweris1.1255
Times20,000,thats$22,510
(asopposedto22,400forsimpleinterest)
2
GMATClub contributingtoeachotherslearning
Mixtures
MATH:WORDPROBLEMS
14litersofapplejuiceismixedwithcranberry
juice.Iftheresultingmixcontains65%of
cranberryjuice,howmanylitersofthemix
wereproduced?
3
GMATClub contributingtoeachotherslearning
Mixtures|Answers
MATH:WORDPROBLEMS
Mixtureproblemsrequireattentiontodetailsto
boththeinformationgivenandthequestion.
Weknowthatthe14litersofapplejuiceis35%
(100%65%)ofthenewmixture.
ConstructanX:
14liters35%
Xliters100%

14100
35
= 4u liters
4
GMAT Club 2011 35
GMATClub contributingtoeachotherslearning
WorkProblems
MATH:WORDPROBLEMS
Whatistheformulaforawork
problem?
5
GMATClub contributingtoeachotherslearning
WorkProblems|Answers
MATH:WORDPROBLEMS
Thekeytosolvingtheworkproblems,issettingthe
equationcorrectly.Theworkformulaisbasedonthe
principleofworkrates(inverseofthetimeitwould
taketocompletethejob).Theratealmostalwayswill
be
1
tmc (o] hous, duys, ctc)
Formula:SumoftheRatesofWorkers=thecombined
rate

1
w
1
+
1
w
2
=
1
C
;whereWistimerequiredbyworkers1
and2tocompletethejobandCisthetimerequired
forbothworkersworkingtogether(Combined)
6
GMATClub contributingtoeachotherslearning
WorkProblems2
MATH:WORDPROBLEMS
Robertworkingalonecanunloadatruck
in8hours.Doug,ontheotherhand,can
unloadthesametruckin6hours.Ifboth
arehiredtogether,howmuchtimewillit
takeRobertandDougtounloadthe
truckworkingtogether?
7
GMATClub contributingtoeachotherslearning
WorkProblems2|Answers
MATH:WORDPROBLEMS
UsingtheWorkformula:
1
R
+
1

=
1
C

1
8
+
1
6
=
1
C

7
24
=
1
C
C=
24
7
= S.4hours(approximately)
8
GMAT Club 2011 36
GMATClub contributingtoeachotherslearning
GroupProblems
MATH:WORDPROBLEMS
Outof90conferenceattendees,50registered
forthebasicworkshopand60signedupfor
theadvancedworkshop.If20attendeeshave
notsignedupforaworkshopyet,howmany
signedupforbothadvancedandbasic
workshops?
9
GMATClub contributingtoeachotherslearning
GroupProblems|Answers
MATH:WORDPROBLEMS
Thebestandeasiestapproachtosolvingthistype
ofproblemsisusingtheboth/neitherformula
(alternativeoptionisaVenndiagram).
Group1+Group2+Neither Both=Total
50+60+20 Both=90
130 Both=90
Both=40
Thekeyistomemorizetheformulaandone
samplequestionsoyoucanpluginnumberson
thetest
10
GMATClub contributingtoeachotherslearning
GroupProblems2
MATH:WORDPROBLEMS
Theofficeof120issplitbetweenmaleandfemale
employeesattheratioof3:5.If40%ofthe
employeesaremarriedand20ofthemarried
employeesintheofficearemen,howmanyofthe
womenworkingintheofficearesingle?
11
GMATClub contributingtoeachotherslearning
GroupProblems2|Answers
MATH:WORDPROBLEMS
Toanswerthisquestionthefastest,wecanputthis
tabletogether:
Thus20+X=48;X=28.Wecandotherestofthe
mathandfillouttheentiretabletomakesureyour
calculationsarecorrect
Male Female Total
Married 20 X 48
Single X 3 X 75 72
Total 45 75 120
12
GMAT Club 2011 37
GMATClub contributingtoeachotherslearning
Volume/MixtureProblems(Hard)
MATH:WORDPROBLEMS
Ifafarmersells15ofhischickens,hisstockoffeedwill
lastfor4moredaysthanplanned,butifhebuys20
morechickens,hewillrunoutoffeed3daysearlier
thanplanned.Ifnochickensaresoldorbought,the
farmerwillbeexactlyonschedule.Howmanychickens
doesthefarmerhave?
12
24
48
55
60
13
GMATClub contributingtoeachotherslearning
Volume/MixtureProblems|Answers
MATH:WORDPROBLEMS
Veryhardproblem.Severalsolutionsexist;thisone
isprobablynotthemostcorrectbutthequickest:
LetXbethenumberofchickensandYbethedays
theycansurviveonthecurrentfeed:
(x15)(y+4)=(x+20)(y3)
xy +4x -1Sy -6u = xy -Sx +2uy -6u
4x -1Sy = 2uy -Sx
7x = SSy or x = Sy
Solvingthefirstequation,wegetx=60andy=12
14
GMATClub contributingtoeachotherslearning
CountingProblems(UltraHard)
MATH:WORDPROBLEMS
Inagameofchess,themovesofwhitesandblacks
alternatewithwhiteshavingthefirstmove.During
achesstournament,whiteshavemade2319
movesaltogetherwhileblackshavemade2315
moves.Ifinanygamethesidethatmadethelast
movedidnotlose,whichofthefollowingcanbe
trueaboutthetournament?
I.Blackslost5games
II.Blackswonmoregamesthanwhites
III.Allgamesendedinadraw
15
GMATClub contributingtoeachotherslearning
CountingProblems|Answers
MATH:WORDPROBLEMS
Fromthestemitfollowsthattherewereonly4gamesin
whichwhiteshadthelastmove.These4gameswere
responsibleforthedifferenceinthetotalnumberofmoves
madebywhitesandblacksduringthetournament.We
knowthatthese4gameswerenotwonbyblacks(butthey
couldwellhaveendedinadraw).Alltheothergames
couldhavebeenwonbyblacksorendedinadraw.Thus,
scenariosIIandIIIarepossible.
ScenarioIisimpossible.Itmeansthattherewereatleast5
gamesinwhichwhiteshadthelastmove.Ifthisweretrue
thenthedifferencebetweenthetotalnumberofmovesof
whitesandblacksshouldbeatleast5.Infact,it'sonly4.
16
GMAT Club 2011 38
GMATClub contributingtoeachotherslearning
Distance(UltraHard)
MATH:WORDPROBLEMS
Aswimmermakesaroundtripupanddownthe
riverwhichtakesherXhours.Ifthenextdayshe
swimsthesamedistancewiththesamespeedin
stillwater,whichtakesherYhours,whichofthe
followingstatementsistrue?
X>Y
X<Y
X=Y
X =
1
2

17
GMATClub contributingtoeachotherslearning
Distance|Answers
MATH:WORDPROBLEMS
Picknumbersandthencheckthemagainstthe
options.Take12kmasthedistancetraveled
up/downtheriver,andassumetheswimmer's
speedtobe4km/h;thecurrentbeing2km/h,
whichmeans6km/hdowntheriverand2km/h
uptheriver.Goinguprivertakes2hours,return
journeytakes6,thusatotalof8hours.Instill
water,24kmrequires6hours.ThusX=8andY=6.
Plugtheseintotheanswerchoices.(8>6).
ThecorrectanswerisA
18
GMATClub contributingtoeachotherslearning
Percent
MATH:WORDPROBLEMS
Ifapricewasincreasedbyx%andthen
decreasedbyy%,isthenewpricehigherthan
theoriginal?
x>y
x=1.2y
19
GMATClub contributingtoeachotherslearning
Percent|Answers
MATH:WORDPROBLEMS
LetPdenotetheoriginalprice.
Statement(1)byitselfisinsufficient.Ifxismuchlarger
thany,thenewpriceishigherthantheoriginal.Butifxis
onlymarginallylarger,thenewpriceislower.Forexample,
ifx=20andy=19,thenewpriceisP*1.2*0.81=0.97P<P.
Statement(2)byitselfisinsufficient.Usethesame
reasoning.Ifyislarge,thenewpriceissmall(ify=100,the
newpriceis0).Ifyissmall,thenewpriceishigherthan
theoriginal(ifx=12andy=10,thenewpriceisP*1.12*0.9=
P*1.008>P).
Statements(1)and(2)combinedareinsufficient.Adding
S1toS2providesnonewinformation.
ThecorrectanswerisE.
20
GMAT Club 2011 39
GMATClub contributingtoeachotherslearning
Rate
MATH:WORDPROBLEMS
Acertainbacteriacolonydoublesinsizeeveryday
for20days,atwhichpointitreachesthelimitofits
habitatandcannolongergrow.Iftwobacteria
coloniesstartgrowingsimultaneously,howmany
dayswillittakethemtoreachthehabitatslimit?
6.33
7.5
10
15
19
21
GMATClub contributingtoeachotherslearning
Rate|Answers
MATH:WORDPROBLEMS
Weknowthatthebacteriacolonydoublesinsize
everydayfor20days.Thereforeonthesecondday
itisdoublethesizeofthefirstday,andsoon.
Similarly,onthe20
th
day,itisat100%ofcapacity,
therefore,onthe19
th
day,itwillbeat50%.Since
wehave2colonies,bothwillbeoccupyinghalfof
thehabitatbythe19
th
day.Alternatively:
1colony=x 2
20
;2colonies=
x2
20
2
= x 2
19
ThecorrectanswerisE.19days.
22
GMAT Club 2011 40
GMAT Club contributing to each others learning
Triangles

Sum of angles of ANY triangle equals ?
What can we say about sides of a triangle?
What is the right triangle?
MATH: GEOMETRY
1
GMAT Club contributing to each others learning
Triangles | Answers
Sum of all angles in any triangle is always 180
One side is always smaller than the sum of the
other two and is always greater than the difference
of the other two
A right triangle is the one that has a 90 degree
angle (it has the right angle). A triangle can only
have one angle at 90 egrees since sum of the 3
angles is 180
MATH: GEOMETRY
2
GMAT Club contributing to each others learning
Triangles 2


List all methods for finding an area
of a triangle.
MATH: GEOMETRY
3
GMAT Club contributing to each others learning
Triangles 2 | Answers

1. Area =
1
2

2. Heros formula: ( )( )( )
where a,b,c are sides of a triangle and s is semi-
perimeter =
::
2

3. If you know 2 sides of a triangle but not its height,
you can add an equally sized triangle to create a
square/rectangle/rhombus and find its area (may be
easier). Remember to divide your result by 2.
MATH: GEOMETRY
4
GMAT Club 2011 41
GMAT Club contributing to each others learning
Triangles 3
These are 2 sides of a right triangle, find the third
side:
3, 4, x
6, 8, x
5, 12, x
12, 16, x
7, 24, x

MATH: GEOMETRY
5
GMAT Club contributing to each others learning
Triangles 3 | Answers
GMAT relies on these easy triangles. If you
memorize these combinations, it will save you time
on the Geometry section
3, 4, 5
6, 8, 10
5, 12, 13
12, 16, 20
7, 24, 25
MATH: GEOMETRY
6
GMAT Club contributing to each others learning
Triangles 4


What is the relationship between sides in
a right isosceles triangle?
What is the relationship between
angles?

MATH: GEOMETRY
7
GMAT Club contributing to each others learning
Triangles 4 | Answers
A right isosceles triangle will have angles that are
90, 45, 45 degrees
It will have sides that are x, x, and the hypotenuse
of 2
MATH: GEOMETRY
8
GMAT Club 2011 42
GMAT Club contributing to each others learning
Triangles 5


Name as many properties,
relationships, and formulas you
know about and equilateral triangle
MATH: GEOMETRY
9
GMAT Club contributing to each others learning
Triangles 5 | Answers
All sides are equal
All angles are equal
Area =
2

3
4
where as is a side of a triangle
A height is =
3
2

For a given area, an equilateral triangle has the
smallest perimeter it is the most efficient
triangle
MATH: GEOMETRY
10
GMAT Club contributing to each others learning
Triangles 6
What can you derive from this figure?
(M is the center of a circle)
A
B
C
M
MATH: GEOMETRY
11
GMAT Club contributing to each others learning
Triangles 6 | Answers
AC is the diameter of a circle
=

2
where R is radius
AM = MC = MB
Angle ABC is a right angle
If one of the sides on an inscribed triangle is the
circles diameter, then this is a right triangle
regardless of the position of point B
MATH: GEOMETRY
12
GMAT Club 2011 43
GMAT Club contributing to each others learning
Triangles 7

What is the value of sides in a
30-60-90 triangle?
MATH: GEOMETRY
13
GMAT Club contributing to each others learning
Triangles 7 | Answers
In 30-60-90 triangle, the sides are x, x 3,
and the hypotenuse is 2x (double the
size of the smallest side)

MATH: GEOMETRY
14
GMAT Club contributing to each others learning
Triangles 8 (Ultra Hard)
Is the area of the triangle ABC less than 1?
ABC < 90 degrees
Perimeter of triangle ABC is greater than

4

MATH: GEOMETRY
15
GMAT Club contributing to each others learning
Triangles 8 | Answers
The area of the triangle ABC is
1
2
1

+
1


2
=
Statement (1) by itself is sufficient. In the extreme case
when Angle ABC is right, the triangle BOC is isosceles
and thus
2
=
1

and the area of the triangle ABC is a =


1. If angle ABC is smaller than 90 degrees, then the
area exceeds 1 due to the increase of the height
2
.
Statement (2) by itself is insufficient. As long as a>0,
the perimeter of the triangle ABC is always greater
than
4


The correct answer is A.

MATH: GEOMETRY
16
GMAT Club 2011 44
GMAT Club contributing to each others learning
Circles
Please define the following:
Center - ?
Radius - ?
Diameter - ?
Circumference - ?
Area - ?
Chord - ?
Tangent - ?
Secant - ?
MATH: GEOMETRY
17
GMAT Club contributing to each others learning
Circles | Answers
Center A point inside the circle. All points on the
circle are equidistant from the center
Radius distance between the center and any point on
the circle. It is half the diameter
Diameter a chord passing through the center
Circumference distance around the circle
Area a region enclosed by the circle
Chord a line segment linking any two points on a
circle
Tangent line touching the circle at one point only;
tangent lines are always at 90 degrees to the radius
Secant a line that intersects a circle in 2 points
MATH: GEOMETRY
18
GMAT Club contributing to each others learning
Circles 2

Area of a circle = ?
Length of a circle = ?
= ?
MATH: GEOMETRY
19
GMAT Club contributing to each others learning
Circles 2 | Answers

Area of a circle =
2

Length of a circle = 2
= 3.14 3
22
7

MATH: GEOMETRY
20
GMAT Club 2011 45
GMAT Club contributing to each others learning
Coordinate Geometry

What is the equation of the
slope of a line?
MATH: GEOMETRY
21
GMAT Club contributing to each others learning
Coordinate Geometry | Answers

Slope of a line equation:

=

2

1

2

1


Where x and y are coordinates of
point 1and point 2 on that line.
MATH: GEOMETRY
22
GMAT Club contributing to each others learning
Coordinate Geometry 2


If a line has a negative slope less
than 1 what does it say about the
line?
MATH: GEOMETRY
23
GMAT Club contributing to each others learning
Coordinate Geometry 2 | Answers
Negative slope means line moves from the upper
left hand quarant (Q2) to the bottom right hand
quadrant (Q4) or in simple terms, it is a decreasing
line. Positive slope means the opposite (duh)
Since the slope is less than 1, it is a flat line (as
opposed to steep). Since slope is rise over run, in
this case, there is less rise than run
MATH: GEOMETRY
24
GMAT Club 2011 46
GMAT Club contributing to each others learning
Coordinate Geometry 3
What slopes do Lines A, B, and C have?
Positive/Negative?
Less/Greater than 1?


A
B
C
MATH: GEOMETRY
25
GMAT Club contributing to each others learning
Coordinate Geometry 3 | Answers
Line A
Positive Slope
Slope greater than 1
Line B
Slope is neither
positive or negative
Slope is Zero
Line C
Slope is undefined

MATH: GEOMETRY
26
GMAT Club contributing to each others learning
Coordinate Geometry 4


How to find the X and Y intercepts
of a line?
MATH: GEOMETRY
27
GMAT Club contributing to each others learning
Coordinate Geometry 4 | Answers
Best option is to plug in the values into the
equation of the line
For example, a line is y = mx + b
To find the Y intercept (this is when the line crosses
the Y axis and thus X is zero) solve: y = b
To find X intercept (this is when the line crosses the
X axis and Y is zero) solve: 0 = mx + b
The trick is to use Y = 0 when looking for X
intercept and X = 0 when looking for Y intercept
MATH: GEOMETRY
28
GMAT Club 2011 47
GMAT Club contributing to each others learning
Coordinate Geometry 5


If line M with a slope of
5
9
goes through points A(-5,
-2) and B(4, 3), what is the length of the segment
AB?
MATH: GEOMETRY
29
GMAT Club contributing to each others learning
Coordinate Geometry 5 | Answers
The slope information in this
irrelevant
To find distance between A
and B is calculated using the
Pythagorean Theorem by
drawing a triangle
9
2
+5
2
=
2

81 +25 =
= 106
MATH: GEOMETRY
30
GMAT Club contributing to each others learning
Coordinate Geometry 6


Find the equation of a line passing
through the points A (5,4) and B (2,3)
MATH: GEOMETRY
31
GMAT Club contributing to each others learning
Coordinate Geometry 6 | Answers
A (5,4) and B (6,3)
To find an equation of a line based on two
points, use this formula:
;
1
;
1
=

1
;
2

1
;
2


;4
;5
=
4 ;3
5 ;6
;
;4
;5
=
1
;1

- + 4 = 5
= + 9
MATH: GEOMETRY
32
GMAT Club 2011 48
GMAT Club contributing to each others learning
Coordinate Geometry 7
If lines A and B are perpendicular to each
other, what is the relationship between their
slopes?
A. Inverse
B. Opposite
C. Positive
D. Reciprocal
E. Reciprocal and Negative
MATH: GEOMETRY
33
GMAT Club contributing to each others learning
Coordinate Geometry 7 | Answers
The relationship between
slopes of 2 perpendicular
lines is negative reciprocal

. In other words, the


two lines are perpendicular
if and only if the product of
their slopes is -1.
E.g. 3
1
3
=-1
MATH: GEOMETRY
34
GMAT Club contributing to each others learning
Coordinate Geometry 8
Are the two lines below perpendicular?

MATH: GEOMETRY
35
GMAT Club contributing to each others learning
Coordinate Geometry 8 | Answers
To answer, find the slope of each line and then
check to see if one slope is the negative reciprocal
of the other or if their product equals to -1.
Slope AB =
5;19
9;48
=
;14
;39
=
14
39

Slope CD =
24;4
22;31
=
20
;9
=
20
9

Multiply the slopes:
14
39

20
9
1;
Not Perpendicular
MATH: GEOMETRY
36
GMAT Club 2011 49
GMAT Club contributing to each others learning
Coordinate Geometry 9


What is the point of intersection of two
lines that have the following equations:
y=3x-3 and y=2.3x+4?



MATH: GEOMETRY
37
GMAT Club contributing to each others learning
Coordinate Geometry 9 | Answers
The key to solving the intersection questions is that
at the point of intersection, both lines will have the
same X and Y coordinates.
Thus, if Y coordinates are the same, then we can
put the two equations together: 3x-3 = 2.3x+4
0.7x = 7; x = 10
Now we still need to find the Y intercept. Plug 10
into one of the equations: 3*10-3 = 27
Intersection point: (10, 27)
MATH: GEOMETRY
38
GMAT Club contributing to each others learning
Coordinate Geometry 10 (Hard)
Does the curve
2
+
2
= 16
intersect the Y axis?
1)
2
+
2
> 16
2) = +5
MATH: GEOMETRY
39
GMAT Club contributing to each others learning
Coordinate Geometry 10 | Answers

2
+
2
= 16 is the equation of a circle
centered at with radius 4.
Statement (1) by itself is insufficient. S1 says that the
center of the circle is further than 4 units away from
the origin but it doesn't specify whether the circle is
far enough from the axis not to intersect it.
Statement (2) by itself is sufficient. From S2 it follows
that and thus the center of the circle is at least 5 units
away from the axis. As the radius of the circle is only 4
units, we can conclude that the circle does not
intersect the axis.
The correct answer is B. Statement 2 is sufficient.

MATH: GEOMETRY
40
GMAT Club 2011 50
GMATClub contributingtoeachotherslearning
CoordinateGeometry8
Arethetwolinesbelowperpendicular?
MATH:GEOMETRY
41
GMATClub contributingtoeachotherslearning
CoordinateGeometry8|Answers
Toanswer,findtheslopeofeachlineandthen
checktoseeifoneslopeisthenegativereciprocal
oftheotheroriftheirproductequalsto1.
SlopeAB=
5-19
9-48
=
-14
-39
=
14
39
SlopeCD=
24-4
22-31
=
20
-9
= -
20
9
Multiplytheslopes:
14
39

20
9
= -1;
NotPerpendicular
MATH:GEOMETRY
42
GMATClub contributingtoeachotherslearning
CoordinateGeometry9
Whatisthepointofintersectionoftwo
linesthathavethefollowingequations:
y=3x3andy=2.3x+4?
MATH:GEOMETRY
43
GMATClub contributingtoeachotherslearning
CoordinateGeometry9|Answers
Thekeytosolvingtheintersectionquestionsisthat
atthepointofintersection,bothlineswillhavethe
sameXandYcoordinates.
Thus,ifYcoordinatesarethesame,thenwecan
putthetwoequationstogether:3x3=2.3x+4
0.7x=7;x=10
NowwestillneedtofindtheYintercept.Plug10
intooneoftheequations:3*103=27
Intersectionpoint:(10,27)
MATH:GEOMETRY
44
GMAT Club 2011 51
GMATClub contributingtoeachotherslearning
CoordinateGeometry10(Hard)
Doesthecurve x -o
2
+ y -b
2
= 16
intersecttheYaxis?
1) o
2
+b
2
> 16
2) o = b +S
MATH:GEOMETRY
45
GMATClub contributingtoeachotherslearning
CoordinateGeometry10|Answers
x -o
2
+ y -b
2
= 16 istheequationofacircle
centeredatwithradius4.
Statement(1)byitselfisinsufficient.S1saysthatthe
centerofthecircleisfurtherthan4unitsawayfrom
theoriginbutitdoesn'tspecifywhetherthecircleis
farenoughfromtheaxisnottointersectit.
Statement(2)byitselfissufficient.FromS2itfollows
thatandthusthecenterofthecircleisatleast5units
awayfromtheaxis.Astheradiusofthecircleisonly4
units,wecanconcludethatthecircledoesnot
intersecttheaxis.
ThecorrectanswerisB.Statement2issufficient.
MATH:GEOMETRY
46
GMAT Club 2011 52
GMATClub contributingtoeachotherslearning
Enumeration
Therearethreemarbles:1blue,1grayand1
green.Inhowmanywaysisitpossibletoarrange
marblesinarow?
MATH:PROBABILITY&COMBINATIONS
1
GMATClub contributingtoeachotherslearning
Enumeration|Answers
Solution: Let'swriteoutallpossibleways
Total:6
Enumerationisamethodofcountingallpossiblewaystoarrange
elements.Althoughitisthesimplestmethod,itisoftenthefastest
methodtosolvehardGMATproblemsandisapivotalprincipleforany
othercombinatorialmethod.Infact,combinationandpermutationis
shortcutsforenumeration.Themainideaofenumerationiswritingdown
allpossiblewaysandthencountthem.
MATH:PROBABILITY&COMBINATIONS
2
GMATClub contributingtoeachotherslearning
Enumeration2
Therearethreemarbles:1blue,1grayand1
green.Inhowmanywaysisitpossibleto
arrangemarblesinarowifblueandgreen
marbleshavetobenexttoeachother?
MATH:PROBABILITY&COMBINATIONS
3
GMATClub contributingtoeachotherslearning
Enumeration2|Answers
Solution: Let'swriteoutallpossiblewaysto
arrangemarblesinarawandthenfindonly
arrangementsthatsatisfyquestion'scondition:
Answer:4
MATH:PROBABILITY&COMBINATIONS
4
GMAT Club 2011 53
GMATClub contributingtoeachotherslearning
Enumeration3
Inhowmanywayscan5dressesbe
arrangedinastoredisplay?
MATH:PROBABILITY&COMBINATIONS
5
GMATClub contributingtoeachotherslearning
Enumeration3|Answers
1.Howmanyobjectswecanputat1stplace?5.
2.Howmanyobjectswecanputat2ndplace?4
andsoon:3,2,1
Therefore,thetotalnumberofarrangementsofn
differentobjectsinarowis
N = n n -1 n -2 2 1 = n!
5!=5*4*3*2*1=20*6=120
N!iscalledafactorial.Factorialequalstothe
productofnumbersfromNto1.
MATH:PROBABILITY&COMBINATIONS
6
GMATClub contributingtoeachotherslearning
Enumeration4(UltraHard)
IfN isapositiveinteger,whatisthelastdigit
of1!+2!+.N!?
Nisdivisibleby4

N
2
+1
5
isanoddinteger
MATH:PROBABILITY&COMBINATIONS
7
GMATClub contributingtoeachotherslearning
Enumeration4|Answers
Thisisaveryhardquestionthatrequiresanon
traditionalapproach(assomeofthehardestofficial
GMATquestionsoftendo)
Analyzingfactorials,youwillnoticethatthesumof
factorialswillhave3asthelastdigitifN>3,(starting
with5!,eachsumendswithazerosince5!=120,
6!=720,andsoon.)
S1issufficientsinceweknowN>3andthuswecansay
withcertaintythatlastdigitequalsto3
S2tellsusthatNisnot1or3andiseither2orgreater
than3.Ineithercase,thelastdigitwillbe3
ThecorrectanswerisD
MATH:PROBABILITY&COMBINATIONS
8
GMAT Club 2011 54
GMATClub contributingtoeachotherslearning
Combinations1
Whatisthenumberofpossiblearrangementsof
objectsk fromacollectionofdistinctobjectsn?
MATH:PROBABILITY&COMBINATIONS
9
GMATClub contributingtoeachotherslearning
Combinations1|Answers
Acombinationisanunordered collectionofkobjects
takenfromasetofndistinctobjects.Thenumberof
wayshowwecanchoosekobjectsoutofndistinct
objectsisdenotedas:C
k
n
Totalnumberofarrangementsofn distinctobjectsis
n!
Nowwehavetoexcludeallarrangementsofkobjects
(k!) andremaining(nk) objects((nk)!) astheorderof
chosenk objectsandremained(nk) objectsdoesn't
matter.
C
k
n
=
n!
k! n-k !
MATH:PROBABILITY&COMBINATIONS
10
GMATClub contributingtoeachotherslearning
Combinations2
Whatisthenumberofpossiblearrangementsof
objectsk inacertainorderfromacollectionof
distinctobjectsn?
MATH:PROBABILITY&COMBINATIONS
11
GMATClub contributingtoeachotherslearning
Combinations2|Answers
Apermutationisanordered collectionofkobjects
takenfromasetofndistinctobjects.Thenumber
ofwayshowwecanchoosekobjectsoutofn
distinctobjectsisdenotedas:P
k
n
1.Thetotalnumberofarrangementsofndistinct
objectsisn!
2.Nowwehavetoexcludeallarrangementsof
remaining(nk)!Objects
P
k
n
=
n!
n-k !
MATH:PROBABILITY&COMBINATIONS
12
GMAT Club 2011 55
GMATClub contributingtoeachotherslearning
Combinations3
Whatisthedifferencebetween
combinationsandpermutations?
Whentousewhichformula?
MATH:PROBABILITY&COMBINATIONS
13
GMATClub contributingtoeachotherslearning
Combinations3|Answers
PermutationsformulaP =
n!
n-k !
isusedwhen
sequenceofchoicematters(meaningagroupABC
isdifferentfromBACorCBA).Classicexampleis
choosingnomineesfor3specificpositionsfroma
poolof10candidates
CombinationsformulaC=
n!
k! n-k !
isusedwhen
orderofselectionhasnoimpactandonceasmall
groupisformed,itdoesnotmatterhowthey
arrivedthere.Classicexampleispicking3marbles
fromabagof10
MATH:PROBABILITY&COMBINATIONS
14
GMATClub contributingtoeachotherslearning
Combinations4
Ifsixbusinesspartnersarehavingadinnerat
aroundtable,howmanyseating
arrangementsarepossible?
MATH:PROBABILITY&COMBINATIONS
15
GMATClub contributingtoeachotherslearning
Combinations4|Answers
Thedifferencebetweenplacementinarowand
thatinacircleisfollowing:ifweshiftallobjectby
oneposition,wewillgetdifferentarrangementin
arowbutthesamerelativearrangementina
circle.So,forthenumberofcirculararrangements
ofn objects,insteadofn!,wehave (n1)!
Thus,theansweris5!or120
MATH:PROBABILITY&COMBINATIONS
16
GMAT Club 2011 56
GMATClub contributingtoeachotherslearning
Combinations5
Ifthereare5chairsinaroomandBoband
RachelwanttositsothatBobisalwaysleftof
Rachel,inhowmanywaysthisseating
arrangementbeachieved?
MATH:PROBABILITY&COMBINATIONS
17
GMATClub contributingtoeachotherslearning
Combinations5|Answers
NotethatleftofRachel,doesnotmean
immediatelynexttoRachel,justleftofher.
Thisconditioniscalledsymmetrybecauseit
eliminateshalfofthepossibilities(Rachelcansit
onlyleftorrightofBob).
Therefore,thenumberofwaysthatBobisleftof
Rachelisexactly
1
2
ofallpossiblewaysor
N =
1
2
P
5
2
= 1u
MATH:PROBABILITY&COMBINATIONS
18
GMATClub contributingtoeachotherslearning
Combinations6(UltraHard)
Inhowmanydifferentwayscanagroupof8
peoplebedividedinto4teamsof2people
each?
90
105
168
420
2520
MATH:PROBABILITY&COMBINATIONS
19
GMATClub contributingtoeachotherslearning
Combinations6|Answers
Thesolutiontothisproblemisthenumberof
combinations.Firstwegetoneteamoutof8.The
numberofwaystodothiswouldbeC
8
2
.Thenext
combinationis2outof6orC
4
2
,andsoon.Havingall
fourcombinationsmultiplied,weneedtodividethe
totalnumberbythenumberofwaystheteamscanbe
chosen,sincewearenotinterestediftheteamwith
twocertainpeopleischosenfirst,secondorthird.
Therefore,theanswerisfoundbythefollowing
formula:
C
8
2
C
6
2
C
4
2
C
2
2
4!
= 1uS
ThecorrectanswerisB.
MATH:PROBABILITY&COMBINATIONS
20
GMAT Club 2011 57
GMATClub contributingtoeachotherslearning
Probability1
Whatistheprobabilitythatan
eventn willoccur?
Whatistheprobabilitythatan
eventn willnot occur?
MATH:PROBABILITY&COMBINATIONS
21
GMATClub contributingtoeachotherslearning
Probability1|Answers
Theprobabilitythatanevenn willtakeplaceis
n
N
whereN isthetotalnumberofpossible
occurrences
Theprobabilitythatanevenn willnotoccuristhe
oppositeofitoccurring,so1 -
n
N
or1 p
MATH:PROBABILITY&COMBINATIONS
22
GMATClub contributingtoeachotherslearning
Probability2
WhatistheprobabilityofgettingTails when
flippingacoin?
Whatistheprobabilityofgettinga4whenrollinga
die?
MATH:PROBABILITY&COMBINATIONS
23
GMATClub contributingtoeachotherslearning
Probability2|Answers
TheprobabilityofgettingTailswhenflippingacoin
is
1
2
or50%sincethereare2totalpossibilitiesand
onlyoneoutcomeeachtimethecoinisflipped
Theprobabilityofgettinga4whencastingadieis
1
6
;thereareatotalof6potentialpossibilities
(1,2,3,4,5,6)andonlyonechancetorolloneof
them.
MATH:PROBABILITY&COMBINATIONS
24
GMAT Club 2011 58
GMATClub contributingtoeachotherslearning
Probability3
Abucketcontains10greenand90white
marbles.IfAdamrandomlychoosesamarble,
whatistheprobabilitythatitwillbegreen?
MATH:PROBABILITY&COMBINATIONS
25
GMATClub contributingtoeachotherslearning
Probability3|Answers
Thenumberofgreenmarbles:n=10
Thenumberofallmarbles:N=10+90=100
Probability:
10
100
=
1
10
= 1u%
Thereisoneimportantconceptinproblemswith
marbles/cards/balls.Whenthefirstmarbleisremoved
fromajarandnotreplaced,theprobabilityforthe
secondmarblediffers(
10
100
vs.
10
99
).Whereasincaseof
acoinordicetheprobabilitiesarealwaysthesame(
1
6
and
1
2
).Usually,aproblemexplicitlystates:itisa
problemwithreplacementorwithoutreplacement.
MATH:PROBABILITY&COMBINATIONS
26
GMATClub contributingtoeachotherslearning
Probability4
Ifthereisacoinandadie,whatisthe
probabilityofgettingheadsanda"4
afteroneflipandonetoss?
MATH:PROBABILITY&COMBINATIONS
27
GMATClub contributingtoeachotherslearning
Probability4|Answers
Tossingacoinandrollingadieareindependent
events (occurrenceofoneeventdoesnot
influenceoccurrenceofotherevents).Forn
independenteventstheprobabilityistheproduct
ofallprobabilitiesofindependentevent.
So,theprobabilityofgettingheadsis
1
2
and
probabilityofgettinga"4"is
1
6
.Therefore,the
probabilityofgettingheadsanda"4"is:
1
2

1
6
=
1
12
MATH:PROBABILITY&COMBINATIONS
28
GMAT Club 2011 59
GMATClub contributingtoeachotherslearning
Probability5
Ifthereisa20%chanceofrainonanaverage
day,whatistheprobabilitythatitwillrainon
thefirstdayandwillbesunnyonthesecond?
MATH:PROBABILITY&COMBINATIONS
29
GMATClub contributingtoeachotherslearning
Probability5|Answers
Theprobabilityofrainis0.2;thereforeprobability
ofsunshineisq=1 0.2=0.8.Thisyieldsthatthe
probabilityofrainonthefirstdayandsunshineon
theseconddayis:
P=0.2*0.8=0.16
Note: whenworkingwithpercents,itisimportant
toconvertthemintoadecimalformat(suchas0.2
for20%orafractionformatsuchas
1
5
for20%)
MATH:PROBABILITY&COMBINATIONS
30
GMATClub contributingtoeachotherslearning
Probability6
Therearetwosetsofcardswithnumbers:
{1,3,6,7,8}and{3,5,2}.IfRobertchooses
randomlyonecardfromthefirstsetandone
cardfromthesecondset,whatisthe
probabilityofgettingtwooddnumbers?
MATH:PROBABILITY&COMBINATIONS
31
GMATClub contributingtoeachotherslearning
Probability6|Answers
Thereisatotalof5cardsinthefirstsetand3of
themareodd:{1,3,7}.Therefore,theprobability
ofgettingoddcardoutofthefirstsetis
3
5
.
Thereare3cardsinthesecondsetand2ofthem
areodd:{3,5}.Therefore,theprobabilityofgetting
anoddcardoutofthesecondsetis
2
3
.Finally,the
probabilityofgettingtwooddintegersis:
3
5

2
3
=
2
5
or40%
MATH:PROBABILITY&COMBINATIONS
32
GMAT Club 2011 60
GMATClub contributingtoeachotherslearning
Probability7
IfJessicarollsadie,whatisthe
probabilityofgettingatleasta"3"?
MATH:PROBABILITY&COMBINATIONS
33
GMATClub contributingtoeachotherslearning
Probability7|Answers
Twoeventsaremutuallyexclusiveiftheycannot
occuratthesametime.Fornmutuallyexclusive
eventstheprobabilityisthesumofallprobabilities
ofevents:
P(AorB)=P(A)+P(B)
Thereare4outcomesthatsatisfyourcondition(to
rollatleast3):{3,4,5,6}.Theprobabilityofeach
outcomeis
1
6
.
Theprobabilityofgettingatleasta"3"is:
1
6
+
1
6
+
1
6
+
1
6
=
4
6
=
2
3
MATH:PROBABILITY&COMBINATIONS
34
GMATClub contributingtoeachotherslearning
Probability8
Thereare8employeesincludingBoband
Rachel.If2employeesaretoberandomly
chosentoformacommittee,whatisthe
probabilitythatthecommitteeincludesboth
BobandRachel?
MATH:PROBABILITY&COMBINATIONS
35
GMATClub contributingtoeachotherslearning
Probability8|Answers
Combinatorialapproach:
ThetotalnumberofpossiblecommitteesisC
2
8
=28
ThenumberofpossiblecommitteethatincludesbothBob
andRachelis1
P=
1
28
Probabilityapproach:
TheprobabilityofchoosingBoborRachelasafirstperson
incommitteeis
2
8
.TheprobabilityofchoosingRachelor
Bobasasecondpersonwhenfirstpersonisalreadychosen
is
1
7
.TheprobabilitythatthecommitteeincludesbothBob
andRachelis
1
28
MATH:PROBABILITY&COMBINATIONS
36
GMAT Club 2011 61
GMATClub contributingtoeachotherslearning
Probability8 Part2
Thereare8employeesincludingBoband
Rachel.If2employeesaretoberandomly
chosentoformacommittee,whatisthe
probabilitythatthecommitteeincludesboth
BobandRachel?
MATH:PROBABILITY&COMBINATIONS
37
GMATClub contributingtoeachotherslearning
Probability8 Part2|Answers
ReverseCombinatorialApproach:
Insteadofcountingprobabilityofoccurrenceofcertain
event,sometimesitisbettertocalculatetheprobabilityof
theoppositeandthenuseformulap=1 q.
ThetotalnumberofpossiblecommitteesisC
2
8
=28
Thenumberofpossiblecommitteethatdoesnot includes
bothBobandRachelis:m = C
2
6
+2 C
1
6
where,C
2
6
is
thenumberofcommitteesformedfrom6remaining
people
2 C
1
6
isthenumberofcommitteesformedfromRobor
Rachelandoneoutof6otherpeople
P = 1 -
m
N
= 1 -
C
2
6
+2C
1
6
C
2
8
= 1 -
15+26
28
= 1 -
27
28
=
1
28
MATH:PROBABILITY&COMBINATIONS
38
GMATClub contributingtoeachotherslearning
Probability8 Part3
Thereare8employeesincludingBoband
Rachel.If2employeesaretoberandomly
chosentoformacommittee,whatisthe
probabilitythatthecommitteeincludesboth
BobandRachel?
MATH:PROBABILITY&COMBINATIONS
39
GMATClub contributingtoeachotherslearning
Probability8 Part3|Answers
Reverseprobabilityapproach:
Wecanchooseanyfirstperson.
Then,ifwehaveRachelorBobasthefirstchoice,we
canchooseanyotherpersonoutofthe6remaining
people.
IfwehaveneitherRachelnorBobasfirstchoice,we
canchooseanypersonoutoftheremaining7people.
TheprobabilitythatthecommitteeincludesbothBob
andRachelis:P = 1 -
2
8

6
7
+
6
8
1 = 1 -
27
28
=
1
28

MATH:PROBABILITY&COMBINATIONS
40
GMAT Club 2011 62
GMATClub contributingtoeachotherslearning
Probability9
JuliaandBrainplayagameinwhichJulia
takesaballandifitisgreen,shewins.Ifthe
firstballisnotgreen,shetakesthesecond
ball(withoutreplacingfirst)andshewinsif
thetwoballsarewhiteorifthefirstballis
grayandthesecondballiswhite.Whatisthe
probabilityofJuliawinningifthejarcontains
1gray,2whiteand4greenballs?
MATH:PROBABILITY&COMBINATIONS
41
GMATClub contributingtoeachotherslearning
Probability9|Answers
Sometimes,at700+levelyoumayseecomplex
probabilityproblemsthatincludeconditionsor
restrictions.Forsuchproblemsitcouldbehelpful
todrawaprobabilitytreethatincludeallpossible
outcomesandtheirprobabilities.
Now,Itisprettyobvious:
P =
4
7
+
2
7

1
6
+
1
7

2
6
=
2
3
MATH:PROBABILITY&COMBINATIONS
42
GMAT Club 2011 63
GMAT Club contributing to each others learning
Recommended Math GMAT Books
MATH: ARITHMETIC
Need to start from the beginning?
MGMAT Math Foundations
Need a good level of practice?
Kaplan Math Workbook
Need the most practice?
MGMAT Math Guides or Veritas Prep Math Guides
Quantitative GMAT Official Guide 2
nd
ed
GMAT Total Math
GMAT book reviews: http://gmatclub.com/books
GMAT Club 2011 64
GMAT Club contributing to each others learning
Best Math GMAT Courses
MATH: ARITHMETIC
Kaplan Math Refresher Course - $299 or Free
Get $105 bonus when you sign up here:
http://gmatclub.com/kaplan
Kaplan Quiz Bank 1,000 math questions
Use GMAT Clubs discount to get 50% off. Visit
http://gmatclub.com/qbank for the latest promo code
Knewton full course for only $299
The best discount code plus $105 bonus ($496 off)
http://gmatclub.com/knewton
GMAT Club 2011 65
V ERBA L STRA TEG IES
GMAT Club 2011 66
GMATClub contributingtoeachotherslearning
ContentsofFlashCards
BasicStrategiesandPrinciplesofSentence
Correction,CriticalReasoning,andReading
Comprehensionwithafewexamples
Illustrationoferrorsandrightanswerchoices
throughexamples
2
VERBAL
GMAT Club 2011 67
CriticalReasoning
GMATClub contributingtoeachotherslearning
Contents
MainPoint/MustBeTrue
Weaken
Strengthen
Assumption
ResolvetheParadox
Reasoning
BoldFaced
NumbersandPercents
2
VERBAL
GMATClub contributingtoeachotherslearning
BasicDeconstruction
Step1:Readthequestionstemandcategorizethequestion.
Step2:Readthestimulusandidentifythepremiseandthe
conclusion
Step3:Trytofocusontheconclusionandthinkofanswerchoices
thatmightberight
Step4:Useprocessofeliminationtoruleoutwronganswer
choices.Don'ttrytomakethemfit!
Step5:Makesureanswerchoicemakessense!
3
VERBAL
GMATClub contributingtoeachotherslearning
MainpartsofaCRquestion
Conclusion
Premise Assumption
4
VERBAL
GMAT Club 2011 68
GMATClub contributingtoeachotherslearning
MainpartsofaCRquestion
Conclusion:Thisisthefinalargumentthatthe
authormakes.
Premise:Theseareevidentiarystatementsthat
supporttheconclusion
Assumption:Theseareunstatedpremises,on
whichtheconclusionandsometimesthepremise
reston.
5
VERBAL
GMATClub contributingtoeachotherslearning
Premise&Conclusion
PREMISE CONCLUSION
Supportstheconclusion Answersthe
questionofWhy?
Hasatoneoffinality andconveysthefinal
messageofwhattheauthorissaying
Because Thus
Since Therefore
For/Forthereason Hence
Dueto So
Asindicatedby Asaresultof/Consequently
Furthermore Accordingly
Giventhat Itfollowsthat/Itmustbethat
6
VERBAL
GMATClub contributingtoeachotherslearning
TypeI AscertainConclusion
Thesearequestionswhereweassumethatthe
stimulusistrue andtrytofindanswerchoicesthat
aresupportedbytheconclusion
PossibleQuestionTypes:
1. Inference
2. MainPoint
3. MustbeTrue
7
VERBAL
GMATClub contributingtoeachotherslearning
TypeII Strengthen&Support
Thesearequestionswhereweassumethatthe
givenanswerchoicesaretrue andtrytopickthe
bestonethatwillsupportthestimulus
PossibleQuestionTypes:
1. Strengthen
2. Assumption
8
VERBAL
GMAT Club 2011 69
GMATClub contributingtoeachotherslearning
TypeIII WeakenandHurt
Thisisbasicallytheoppositeoftheabovetypeand
aimstodisprovetheconclusionofthestimulus.
Hencewetaketheanswerchoicetobetrue here
aswell.
PossibleQuestionTypes:
1. Strengthen
2. Assumption
9
VERBAL
GMATClub contributingtoeachotherslearning
MostCommonMistakeTypes
OppositeAnswers
Doestheoppositeofwhattheanswerchoiceis
supposedtodo
Peoplepickthembecausetheymightgetconfused
aboutthequestiontype
ShellGameAnswers
Soundsreallysimilartothestimulusbutdiffersbyjust
therightamounttobeincorrectbutstilllucrative.
Peoplepickthesewhentheyrenotpayingclose
attention
10
VERBAL
GMATClub contributingtoeachotherslearning
OutofScope/IrrelevantAnswers
Talksaboutsomethingcompletelyirrelevanttothe
discussionathand.
Peopletendtopickthesewhentheyrecompletely
unsureofwhattheyresupposedtobedoing
ToneMismatchAnswers
Answersthatdontagreewiththetoneofthepassage
Mightbetoostrongortooweakwhencomparedto
stimulus
MostCommonMistakeTypes
11
VERBAL
GMATClub contributingtoeachotherslearning
MainPoint/MustBeTrue
Whichofthefollowingrepresentsthemainideaoftheparagraph?
Whichofthefollowingcanbeinferredfromtheabove?
CorrectAnswerChoices
Canthisanswerchoicebeprovenorvalidatedbywhatisgivenin
thestimulus?IsthisanswerchoicetruetothestimulusANDthe
mainpointofthepassage,i.e.similartotheconclusion?
Shouldbevalidatedbythestimulus(Stimulusistakentobetrue)
Shouldbethemainpointofthestimulus,notjustapremise(for
MainPointquestions)
Willeitherrestateconclusionorpresentitinadifferentmanner
12
VERBAL
GMAT Club 2011 70
GMATClub contributingtoeachotherslearning
Tobeconsideredforthisyearsmeritscholarshipaward,studentsneedto
haveperfectattendanceanda4.0GPA.Alexistheonlypersonintheclass
whohasa4.0buthehashad5absences.
Theclaimsabove,iftrue,moststronglysupportwhichofthefollowing
conclusions?
A. Nostudentatthisschoolhasperfectattendancefortheyear
B. Somestudentsatthisschoolwhodidnothavea4.0alsodidnothave
perfectattendance
C. Alexistheonlystudentwhocouldbeconsideredfortheaward
D. Nostudentatthisschoolqualifiesfortheawardthisyear
E. Manystudentshaveachievedperfectattendancebutnever4.0GPAs.
ABCDE
13
VERBAL
GMATClub contributingtoeachotherslearning
A:Exaggeration.
B:Possible,butnotnecessary
C:Thestimulusclearlysaysthatyouneedboth
perfectattendanceand4.0GPA.
D:Thisistrue.IfAlexistheonlyonewhohasa
4.0andhedoesnthaveaperfectattendance,it
meanstheentireschoolisbarredfrom
qualifyingfortheaward.Correctchoice.
E: Outofscopeanswer.
ABCDE
14
VERBAL
GMATClub contributingtoeachotherslearning
Weaken
Whichofthefollowing,iftrue,callsintoquestionthevalidityofthe
argument?
Whichofthefollowingmostseriouslyunderminestheconclusion?
CorrectAnswerChoices
Doesthisanswerchoicebreakdowncausality?Doesitgivean
alternatecause,showthatthecauseeffectrelationshipisnon
existentorreversed?
Answerchoiceshouldbreakdownstructureofstimulus(Answeris
takentobetrue)
Couldbeinrelationtoagrossgeneralization,awrongconclusion
orincorrecthypothesisfromfacts.
15
VERBAL
GMATClub contributingtoeachotherslearning
16
VERBAL
GMAT Club 2011 71
GMATClub contributingtoeachotherslearning
Thereare350brandsofcellphonesinthemarkettoday.However,ourstore
onlystocksthetop10brands.Inordertoincreaseoursales,weplanto
increasethesizeofourinventorytothetop50brands.
Whichofthefollowing,iftrue,pointsoutamajorflawintheplanabove?
A. Thecapabilitiesofthetopfivecellphonesarealmostthesame,withno
brandhavingconsistentsuperiorityinallrespects.
B. Thetop8brandsaccountforalmostallthecellphonessold
C. Asusersgetmoresophisticated,theywanttotryoutthelesserknown
brandswhichmightoffersomeothervaluetothem.
D. Lesspopularbrandsprovidelittleprofittothestorebecausetheyhaveto
bediscountedtobesold
E. Theleadingbrandsarenowlosingsalestolesspopularbrandsthatoffer
similarfeaturesforalowercost
ABCDE
17
VERBAL
GMATClub contributingtoeachotherslearning
A:Irrelevant.Doesthisaffectprofitmarginsforthe
storeiftheyweretoincreaseinventory?No
B:Thismeansthatthestorealreadyhasthebrands
thatsellthemost.Increasinginventorywillhave
littleeffectonprofitmargins.CorrectAnswer.
C:Thisalmoststrengthenstheargument.
D:Thistellsyouthatthelesserbrandswillprovide
lesserprofit.Butlessisarelativeterm.Thisisan
unclearstatement.
E: Thisalmoststrengthenstheargumentbyfavoring
anincreaseininventory.Henceincorrect.
ABCDE
18
VERBAL
GMATClub contributingtoeachotherslearning
Strengthen
Whichofthefollowing,iftrue,strengthenstheargumentthemost?
Whichofthefollowing,iftrue,wouldmostsignificantlyhelpprovethe
scientistshypothesis?
CorrectAnswerChoices
Doesthisanswerchoicereinforcetheconclusiondirectly?Doesit
validateanassumptionorruleoutadiscrepancy?Doesithelpestablish
causality?
Answerchoiceshouldstrengthenstructureofstimulus(Answeristaken
tobetrue)
Needstodirectlystrengthenconclusionbybridginglogicalgaps,
validatingreasonsorassumptionsorfindingmissinglinks.Ifitsnota
directstrengthening,moveon!Donttrytomaketheconnections!
19
VERBAL
GMATClub contributingtoeachotherslearning
20
VERBAL
GMAT Club 2011 72
GMATClub contributingtoeachotherslearning
Recently,severalcompanieshavewithdrawntheiradsfromMagazineA,becausethe
editorialboardofthemagazinehaddecidedtochangetheimagethatthemagazine
portraysfromoneoffamilyvaluestooneconcernedmorewithsexandviolence.Surely
thisindicatesthatthedecisionmakersinadvertisingagenciesdostillhaveasenseofmoral
proprietythatoccasionallydrivestheiractions.
Whichofthefollowing,iftrue,wouldstrengthenthisconclusion?
A)Theadvertisersregularlyreviewtheplacementoftheiradvertisements.
B)Itisarareeventforseveraladvertiserstowithdrawalltheiradvertisements
simultaneouslyfromapublication.
C)Theadvertisers,whenquestioned,admittedthattheirclientswouldloserevenueasa
resultoftheadvertisementsbeingwithdrawn.
D)Theadvertisersallplacednewadvertisementswithotherpublicationsthatemphasised
familyvalues.
E)AsurveyofthereadershipofMagazineXsuggestedthatthemajorityofthereadership
thinkthatthestandardofthemagazine'scontentshadfailedsinceitstransformation.
QuestionfromGMATClub(95810)

ABCDE
21
VERBAL
GMATClub contributingtoeachotherslearning
A:Thisisirrelevanttothequestionofmoral
propriety.
B:Thisdoesntnecessarilypointtomoral
proprietydirectly.Dontmakeunnecessary
connections!
C:Onceagain,nocorrelationtowhatweretalking
about
D:Ifthisistrue,thenitshowsthattheagencies
careaboutwheretheiradsgo.Hencecorrect.
E: Thisisaboutthemagazine.Nottheadvertisers.
ABCDE
22
VERBAL
GMATClub contributingtoeachotherslearning
Assumption
Theauthorassumeswhichofthefollowinginsayingthat
Theargumentcannotbetrueunlesswhichofthefollowing
statementsareassumed?
CorrectAnswerChoices
Supporter:Linksunrelatedelementsinthestimulusandfillsin
logicalgaps
Defender:Eliminatesthealternativesandanychoicesthatmight
weakentheconclusion.
AssumptionNegationTechnique:Narrowitdowntothefinal
answerchoicesandthennegatethem themainmodifierora
verb(suchthatthemeaningofthesentenceisreversed)andifthe
negatedchoiceweakenstheconclusion,theanswerisright.
23
VERBAL
GMATClub contributingtoeachotherslearning
24
VERBAL
GMAT Club 2011 73
GMATClub contributingtoeachotherslearning
Intheory,itspossiblethatbacteriadevelopedonMarsearlyinitshistoryandsome
werecarriedtoEarthbyameteorite.However,strainsofbacteriafromdifferent
planetswouldprobablyhavesubstantialdifferencesinproteinstructurethatwould
persistovertime,andnotwobacterialstrainsonEartharedifferentenoughtohave
arisenondifferentplanets.So,evenifbacteriadidarriveonEarthfromMars,they
musthavediedout.
Theargumentismostvulnerabletowhichofthefollowingcriticisms?
A.ItfailstoestablishwhetherbacteriaactuallydevelopedonMars.
B.ItfailstoestablishhowlikelyitisthatMartianbacteriaweretransportedtoEarth.
C.Itfailstoconsiderwhetherthereweremeansotherthanmeteoritesbywhich
MartianbacteriacouldhavebeencarriedtoEarth.
D.ItfailstoconsiderwhetherallbacterianowonEarthcouldhavearisenfrom
transportedMartianbacteria.
E.Itfailstoconsiderwhethertherecouldhavebeenstrainsofbacteriathat
originatedonEarthandlaterdiedout.
QuestionfromGMATClub(80726)
ABCDE
25
VERBAL
GMATClub contributingtoeachotherslearning
A:Thisisirrelevanttotheargumentthatstatesthat
evenifbacteriacamefromMars,theymusthave
diedout.
B:OutofScope!
C:Again,thisdoesnttalkaboutbacteriastrainsdying
out
D:Ifthisistrue,thenthereneednothavetobeany
significantdifferencebetweenthestrainsfoundon
Earth,sincetheyalloriginatedfromMars.
E: Wearenotconcernedaboutthis.
ABCDE
26
VERBAL
GMATClub contributingtoeachotherslearning
ResolvetheParadox
Whichofthefollowing,iftrue,helpsexplaintheparadoxabove?
Whichofthefollowing,iftrue,helpsexplaintheapparent
discrepancyintheargument?
CorrectAnswerChoices
ActiveResolution:Donttrytodisprovethestimulus,takeitasa
given.
Doestheanswerchoiceaddressthefacts?Theanswerchoice
MUSTconformtothestimulus
TheanswershouldaddressBOTHsidesoftheparadox,and
resolveit.Itshouldntstrengthentheparadox.
27
VERBAL
GMATClub contributingtoeachotherslearning
28
VERBAL
GMAT Club 2011 74
GMATClub contributingtoeachotherslearning
AseveredroughtcanlessenthetotalamountofgovernmentaidthatUSfarmersreceive
asagroup.Thegovernmentpaysfarmerstheamount,ifany,bywhichthemarketpriceat
whichcropsareactuallysoldfallsshortofapresettargetpriceperbushelforthecrops.
Thedroughtof1983,forexample,causedfarmprogrampaymentstodropby$10billion.
Giventheinformationabove,whichofthefollowing,iftrue,bestexplainswhythe
droughtof1983resultedinareductioninfarmprogrampayments?
A. Priortothedroughtof1983,thegovernmentraisedthetargetpriceforcropsinorderto
aidfarmersinreducingtheirdebtloads.
B. Duetothedroughtof1983,UnitedStatesfarmersexportedlessfoodin1983thaninthe
precedingyear.
C. Duetothedroughtof1983,UnitedStatesfarmershadsmallerharvestsandthusreceived
ahighermarketpriceforthe1983cropthanforthelargercropoftheprecedingyear.
D. Duetothedroughtof1983,UnitedStatesfarmersplannedtoplantsmallercropsin1984
thantheyhadin1983.
E. Despitethedroughtof1983,retailpricesforfooddidnotincreasesignificantlybetween
1982and1983.QuestionfromGMATClub(80726)
ABCDE
29
VERBAL
GMATClub contributingtoeachotherslearning
A:Strengthenstheparadox.
AmountPaid=Target Market.Wecantsay
anythingunlessweknowwhathappenedtothe
marketprice.
B:Exportsareirrelevanthere.
C:Thistalksaboutthehighermarketprices.Thiscould
explaintheparadoxbecausetarget marketwill
becomelesserifmarketbecomeshigher
D:Sizeofcropsisirrelevant.
E: Between19821983?Retailprice?OutofScope!
ABCDE
30
VERBAL
GMATClub contributingtoeachotherslearning
Reasoning
Whichofthefollowingmostaccuratelydescribestheauthors
methodofdefendingthecase?
Whichofthefollowingismostparalleltotheargumentinlogical
structure?
CorrectAnswerChoices
Shouldfollowthesamepatternofreasoning.Focusonthelogic,
notthecontentoftheargument!
Answerchoicemustbeadescriptionoftheauthorslogic!
Answerchoiceshouldnotbeinconsistentwiththestimulus,even
ifthestimulushasflawedlogic.Wearenotaskedtoevaluatethe
logicofthestimulus;wearemerelyaskedtofindananswerthat
followsthesamepattern rightornot.
31
VERBAL
GMATClub contributingtoeachotherslearning
32
VERBAL
GMAT Club 2011 75
GMATClub contributingtoeachotherslearning
Shakespearewastheworld'sgreatestplaywright,whichmeansthathemusthavewrittentheworld's
greatestplays.Thefactthathewrotethegreatestplaysintheworldonlygoestoprovethatheisthe
world'sgreatestplaywright.
Whichofthefollowingstatementsbestmatchestheargumentshownabove?
A. Dr.SmithistheonlymemberofthetwentystrongEnglishdepartmentwhothinksthatByronwrote
greaterpoetrythanKeats.ThereforeByronwasnotasgoodapoetasKeatswas.
B. Amiscarriageofjusticeoccurswhenaninnocentpersonissenttoprison.However,thereareno
miscarriagesofjusticebecausepeopleinprisonareguilty.Otherwisetheywouldnothavebeensent
toprison.
C. Thepresenceofenlargedwhitecorpusclesinthebloodwouldindicatethatthepatientmustbe
sufferingfromdiseaseX.However,hisbloodsamplecontainsnoenlargedwhitecorpuscles,sohe
cannotbesufferingfromdiseaseX.
D. Blairdenieshavingcommittedtherobbery,buthasimplicatedBrown.Brownontheotherhand,
claimsthathewaselsewhereatthetime,andpointsthefingersquarelybackatBlair.
E. Ibelievethatthepresenceofthetartarsaucehelpstoaccentuatetheflavouroftheother
ingredients,buttheyinturnreleasechemicalswhichtendtoneutralisethetartarsauce.Therefore
thedishismuchlessspicythanpuretartarsaucewouldbe.
QuestionfromGMATClub(92697)
ABCDE
33
VERBAL
GMATClub contributingtoeachotherslearning
A:Thiscallsformakingajudgmentbasedon
someonesopinion.Nottrue.
B: Thisistrue.Theargumentinthestimulusisa
circularargumentthatsaysAprovesBandthen
usesBtoproveA.Asimilarpatternexistshere.
C:Thishasanegationeffectinthereasoning.Not
relevant.
D:Shellgameanswer.Mightseemlikecircularlogic,
butitsactuallyjustacircularsequenceofevents.
E: Somekindofcontradictoryreasoning.Notrelevant.
ABCDE
34
VERBAL
GMATClub contributingtoeachotherslearning
BoldFacedQuestions
Thetwoboldfacedsentencesplaywhichofthefollowing
roles?
CorrectAnswerChoices
Shouldaddressboththeboldfacedportions,notjustone.
Willbetruetothestimulusandmimicthereasoningin
thestimulus!
Answerchoiceshouldnotbeinconsistentwiththe
stimulus,anditshouldfollowthesamelogicalpatternin
ordertoarriveattheargumentofthestimulus..
35
VERBAL
GMATClub contributingtoeachotherslearning
36
VERBAL
GMAT Club 2011 76
GMATClub contributingtoeachotherslearning
AlthoughtheearliestsurvivingGreekinscriptionswritteninanalphabetdatefromtheeighthcentury
B.C.,astrongcasecanbemadethattheGreeksactuallyadoptedalphabeticwritingatleasttwo
centuriesearlier. Significantly,thetextoftheseearliestsurvivingGreekinscriptionssometimesruns
fromrighttoleftandsometimesfromlefttoright.Now,theGreekslearnedalphabeticwritingfrom
thePhoenicians,andintheprocesstheywouldsurelyhaveadoptedwhateverconventionthe
Phoenicianswerethenusingwithrespecttothedirectionofwriting. Originally,Phoenicianwriting
ranineitherdirection,butbytheeighthcenturyB.C.ithadbeenconsistentlywrittenfromrightto
leftforabouttwocenturies.
Intheargumentgiven,thetwoportionsinboldfaceplaywhichofthefollowingroles?
A.Thefirstisthepositionthattheargumentseekstoestablish;thesecondreportsadiscoverythat
hasbeenusedtosupportapositionthattheargumentopposes.
B.Thefirstisthepositionthattheargumentseekstoestablish;thesecondpresentsanassumption
onwhichtheargumentrelies.
C.Thefirstpresentsevidencethatisusedinsupportofthepositionthattheargumentseeksto
establish;thesecondpresentsanassumptiononwhichtheargumentrelies.
D.Thefirstisanobjectionraisedagainstapositionthattheargumentopposes;thesecondisthe
positionthattheargumentseekstoestablish.
E.Thefirstisanobjectionraisedagainstapositionthattheargumentopposes;thesecondis
evidencethathasbeenusedtosupportthatposition.
QuestionfromGMATClub(98749)
ABCDE
37
VERBAL
GMATClub contributingtoeachotherslearning
A:Thesecondboldfacedportiondoesntrepresenta
discovery;itsanassumption
B: Thisistrue.Thefirstpartisaclausethestimulus
wantstoproveandthesecondisanassumption
C:Thefirstboldfacedportiondoesntobjecttoany
statement.
D:Thefirstboldfacedportiondoesntobjecttoany
statement.
E: Thefirstboldfacedportiondoesntobjecttoany
statement.
ABCDE
38
VERBAL
GMATClub contributingtoeachotherslearning
Numbers&Percents
Larger numbers doesnt mean larger percentage
and smaller number doesnt mean smaller
percentage. For example,
Increase in percentage is not the same as saying
there is an increase in absolute numbers. This
could merely be the result of a change in the
group size. For example, 5% of 100 > 10% of 50.
39
VERBAL
GMATClub contributingtoeachotherslearning
40
VERBAL
GMAT Club 2011 77
GMATClub contributingtoeachotherslearning
ArecentsurveyofallautoaccidentvictimsinDoleCountyfoundthat,oftheseverelyinjured
driversandfrontseatpassengers,80percentwerenotwearingseatbeltsatthetimeoftheir
accidents.Thisindicatesthat,bywearingseatbelts,driversandfrontseatpassengerscangreatly
reducetheirriskofbeingseverelyinjurediftheyareinanautoaccident.
Theconclusionaboveisnotproperlydrawnunlesswhichofthefollowingistrue?
A. Ofallthedriversandfrontseatpassengersinthesurvey,morethan20percentwerewearingseat
beltsatthetimeoftheiraccidents.
B. Considerablymorethan20percentofdriversandfrontseatpassengersinDoleCountyalways
wearseatbeltswhentravellingbycar.
C. Moredriversandfrontseatpassengersinthesurveythanrearseatpassengerswereveryseverely
injured.
D. Morethanhalfofthedriversandfrontseatpassengersinthesurveywerenotwearingseatbelts
atthetimeoftheiraccidents.
E. MostoftheautoaccidentsreportedtopoliceinDoleCountydonotinvolveanyseriousinjury.
QuestionfromGMATClub(88036)
ABCDE
41
VERBAL
GMATClub contributingtoeachotherslearning
A:Thisisatrickyquestion.Ouraimistoproveacorrelation.Letssay100
peoplewereseverelyinjuredand100werenot.Outofthe100
severelyinjured,80didntwearseatbelts.
Probabilityofapersonnotwearingseatbelttogetinjured=
Probabilityofapersonwearingseatbelttogetinjured=
B:Doesntestablishthecorrelationbetweenwhatsbeingsaid.Hence
incorrect.
C:Thisdoesntevengiveusarelativeindication.Incorrect.
D:Thistellsusaboutthenumberofpeoplewhowerenotwearingseat
belts,butnotaboutthenumberofpeoplewhowereinjured.
E: Completelyirrelevanttothescopeofthediscussion.
ABCDE
42
VERBAL
ReadingComprehension
GMATClub contributingtoeachotherslearning
Contents
GlobalQuestions
MainPoint/PrimaryPurpose
PassageOrganization
AuthorsPerspective/PassageTone
LocalQuestions
SpecificReference
Function
StrengthenandWeaken
ParallelReasoningQuestion
44
VERBAL
GMAT Club 2011 78
GMATClub contributingtoeachotherslearning
FourQuestionsforRC
Readfromgeneraltospecificatthreelevels.Changeyourreading
strategy,notyourreadingspeed.Answerthefollowingquestions.
Why?MainPointofthepassage.
How?Structureofthepassage Introduction,Exampleand
CounterExample.Andsoon.
What?Whatisbeingsaid?(MainPointofIndividualParagraphs)
WhatTone?Makesureanswerchoicemakessense!
45
VERBAL
GMATClub contributingtoeachotherslearning
CONTINUATIONOFOLDIDEAS INTRODUCTION OFNEWIDEAS
Continues elaboratingonanideathats
alreadybeenpresented
Introducesanothernewidea, perhaps
tocontrastsomethingpresented.
Furthermore However
ForInstance But
ForExample Nevertheless
Additionally Yet
Similarly InContrast
Indeed Although
Infact Still
Newvs.ExistingIdeas
46
VERBAL
GMATClub contributingtoeachotherslearning
CommonIndicators
VeryCommonQuestionType
Primarygoalofreadingpassage Findthemain
point!
Main PointorStrongPurpose
47
VERBAL
GMATClub contributingtoeachotherslearning
VeryCommonDistraction
Dontfocusonthedifficultyofthewordsor
terminologies
Instead,choosetofocusontheunderlying
meaningofwhatitssaying
DifficultWords,Phrases&Concepts
CommonIndicators
48
VERBAL
GMAT Club 2011 79
GMATClub contributingtoeachotherslearning
CommonIndicators
Paycloseattention!
Dontmemorize!
Verycommonquestionindicator.Makeamental
noteofwherethelistoccurs,soyoucanreturnto
it,ifnecessary.
ListofThings/Enumerations
49
VERBAL
GMATClub contributingtoeachotherslearning
Ifauthoritiesarementioned,thinkabouthowandwhy
thisauthoritativeremarkisnecessary.
Mightrepresentconflictingviewpointsorideas.
Makenoteofwhateachauthoritysaysandwhytheir
authorityisrelevant.
Very,veryimportant,andveryeasytomisinterpret!
Reference withAuthority
CommonIndicators
50
VERBAL
GMATClub contributingtoeachotherslearning
Matchthecorrectdateswiththeevents
mentioned
Perhaps,makeanoteofthedatesandashort
handversionoftheeventonyournotepad.
DatesandNumbers
CommonIndicators
51
VERBAL
GMATClub contributingtoeachotherslearning
CommonIndicators
Ideasthatarementionedmorethanonceinthe
passage.
Evenifthequestioncitesalinenumberforoneof
thereferences,theanswerchoicemightdrawon
thesameideamentionedelsewhere,oranother
partofthepassage.
HiddenReferences
52
VERBAL
GMAT Club 2011 80
GMATClub contributingtoeachotherslearning
CommonIndicators
Ifseveralviewpointsarepresentedinthepassage,
makenoteofeachpointandwhossayingit/why
itsbeingsaid.
Understandingofthesecounterexamplesorviews
areveryimportant!Theywillbeindicatedby
wordssuchasHoweverorIncontrast
Contrasting Views
53
VERBAL
GMATClub contributingtoeachotherslearning
Commonwhenthepassageisofscientificnature
Makeanoteofthedefinitionandexpecttobe
questionedaboutyourunderstandingofthe
definition
Definitions
CommonIndicators
54
VERBAL
GMATClub contributingtoeachotherslearning
Type
Global Local
BasicQuestionTypes
55
VERBAL
GMATClub contributingtoeachotherslearning
QuestionTypes
MainPoint/PrimaryPurpose
PassageOrganization
AuthorsPerspective/PassageTone
Global BroadQuestions
56
VERBAL
GMAT Club 2011 81
GMATClub contributingtoeachotherslearning
BroadQuestions
RepresentsCoreIdeas
Willaskaboutthebroadermeaningofthe
passage,andwhatitseekstoconvey.
MainPoint/Primary Purpose
57
VERBAL
GMATClub contributingtoeachotherslearning
Thiswillaskaboutthestructureofapassage
Forinstance,thestructuremightbesomething
likethis:
Introduction
Example
CounterExample
Passage Organization
BroadQuestions
58
VERBAL
GMATClub contributingtoeachotherslearning
BroadQuestions
Thesequestionsaskyoutoreflectonanauthors
perspective
Understandwhattheauthoristryingtosay,and
whereheorshestandswithrespecttotheviews
presented.
Istheauthoraggressive?Oristheauthordocile?
Whatisthetoneofthemessageconveyed?
AuthorsPerspective/ToneofPassage
59
VERBAL
GMATClub contributingtoeachotherslearning
QuestionTypes
SpecificReference
Function
StrengthenandWeaken
ParallelReasoningQuestion
Local SpecificQuestions(FourTypes)
60
VERBAL
GMAT Club 2011 82
GMATClub contributingtoeachotherslearning
LocalQuestions
Willrefertoaspecificlineorparagraphinthe
passageandaskforaquestionrelatingtothat.
Mightinvolvecrossreferencingwithother
relevantinformationpresentedelsewhereinthe
passage
SpecificReference
61
VERBAL
GMATClub contributingtoeachotherslearning
LocalQuestions
Questionsaboutwhatapieceofthepassage
eitheraparagraph,alineorevenawordistrying
toaccomplishwithrespecttothebroaderscope
ofthepassage
Trytoinfersomethingaboutthecontextandtone
ofthespecificreference,andaskyourself
Whatdoesthislinedo?
Function
62
VERBAL
GMATClub contributingtoeachotherslearning
LocalQuestions
SimilartoCriticalReasoningQuestionsofthe
sametype.
Therequiredanswerwilleitherhelporhurtthe
conclusiveviewpointpresentedbytheauthor,
i.e.themainpoint
Assumetheanswerchoicesgiventobetrue.
Strengthen/Weaken
63
VERBAL
GMATClub contributingtoeachotherslearning
LocalQuestions
Again,similartoCriticalReasoningQuestionofthe
sametype
Willasktoidentifyanaction,amonggivenactions,or
aviewpointthatmimicsthereasoningthatthe
authorfollows.
Identifyingthestructureofthepassageandthe
structureofalinewithinapassageasnecessary
wouldgreatlyhelp!
Parallel Reasoning
64
VERBAL
GMAT Club 2011 83
GMATClub contributingtoeachotherslearning
RCinaNutshell
Therightmentality:ThepassageWILLbe
intentionallyconfusing.Getusedtoit!
Awarenessofcontent:Thepassagesmightbe
fromhumanities,socialsciencesorsciences.
Dontgetboggeddownbyonekindorget
excitedaboutanother
ReadingPattern:GeneraltoSpecific
65
VERBAL
GMATClub contributingtoeachotherslearning
RCinaNutshell
Understandquestiontypes:GlobalorLocal
Prephrase:Framearoughanswerbeforeyou
pickanswerchoices!
ProcessofElimination:Dontkeepananswer
thatyouhavetomakeconnectionsfor.
66
VERBAL
SentenceCorrection
GMATClub contributingtoeachotherslearning
Contents
SubjectVerbAgreement
VerbTenseErrors
NounAgreement
Pronouns
Modifiers
Comparisons
Parallelism
68
VERBAL
GMAT Club 2011 84
GMATClub contributingtoeachotherslearning
DeconstructingSC
Step1:Readthequestionstemandthinkofpossibleerrorsinthe
sentence,subjectverbagreement,tensemismatchetc.
Step2:Readtheanswerchoiceandsplititintotwogroupsbasedon
overallstructure.
Step3:Oneofthegroupswillcontainanerror.Eliminatethegroup
andresplitthenextgroup.
Step4:Useprocessofeliminationtoruleoutwronganswerchoices.
Don'ttrytomakethemfit!
Step5:Makesuretheanswerchoicemakessense!
69
VERBAL
GMATClub contributingtoeachotherslearning
Grammar Thesentencehastoadheretotherules
ofgrammarfollowedbyStandardEnglish.
Meaning Thesentencehastohavearelevant
meaningandconveyitproperly
Concision Ifwhatisbeingsaidcanbesaidinfive
wordsinsteadoften,thentheformerispreferred
(Activevs.Passivevoice,forexample)
WhatsTested?
ThreeQuestionTypesYouWillSee
70
VERBAL
GMATClub contributingtoeachotherslearning
Spelling TheGMATwillnottestyouonyour
knowledgeofspellings
Punctuation Addingacommaoranapostrophe
andsimilarthingswillnotbetested.Semicolons,
however,aretested.
Capitalization TheGMATdoesnttestyouon
yourknowledgeofcapitalizationeither.
Andwhatsnot?
ThreeQuestionTypesYouWontSee
71
VERBAL
GMATClub contributingtoeachotherslearning
Thisdealswiththeissueofplurality.
Singularsubjectsmustusesingularverbs.For
example:Hewas eating.
PluralVerbsmustusepluralverbs.Forexample:
Theelephantswere walking.
ErrorsTested
SubjectVerbAgreement
72
VERBAL
GMAT Club 2011 85
GMATClub contributingtoeachotherslearning
Removetheadditionalinformationandreadthe
sentencewithoutthem.
BarelyseventeenandleadingtheFrencharmywearing
amansarmor,JoanofArc,anilliteratepeasantgirl
fromtheFrenchcountryside,brokethesevenmonth
oldseizeofOrleansinninedays.
Readingthesentencewithoutthatpartwehave:
BarelyseventeenandleadingtheFrencharmywearing
amansarmor,JoanofArc,brokethesevenmonthold
seizeofOrleansinninedays.
Trap1:
Phrasesbetweensubject andverb
73
VERBAL
GMATClub contributingtoeachotherslearning
Ifthereareexpletives,thencheckforsubjectverb
agreement,byrearrangingthesentence.
Somecommonexpletives:
There
Here
It
Or
Trap2: SubjectFollows Verb
74
VERBAL
GMATClub contributingtoeachotherslearning
Ifthereismorethanonenounortheusageofa
pronouninasentence,thenthesubjectverb
agreementMUSTbeconsistent!!
TwoExceptions:
Conjunctions(OR,NOR) AlwaysSINGULAR
UsageofEACHorEVERY AlwaysSINGULAR
Trap3: MultipleNounsorPronouns
75
VERBAL
GMATClub contributingtoeachotherslearning
Pronounslikeall,any,more,most,somebody,nobody
andsoon.
Pluralityisbasedonwhattheindefinitepronounis
referringto!(Thisistheantecedent)
Exceptions:
Each,Either,Neither,Every,Number Alwayssingular
irrespectiveofwhatfollows!
Example:AnumberofreasonsPlural!Thenumber of
cars Singular!("the" is singular and "a" is plural)
Trap4: Indefinite Pronouns
76
VERBAL
GMAT Club 2011 86
GMATClub contributingtoeachotherslearning
Tense
Past
Present
Future
Forms
Simple
Perfect
Progressive
PerfectProgressive
ErrorsTested
VerbTense
77
VERBAL
GMATClub contributingtoeachotherslearning
IncorrectVerbTense
ShiftinVerbTense
VerbVoice
VerbTense SubTypes
ErrorsTested
78
VERBAL
GMATClub contributingtoeachotherslearning
Thenumberofnounsmustbeconsistentwithwhat
theyarereferencing.
Incorrect: MattandDavebelievedthattheirhard
workintheirengineeringclasswillhelpthemrealize
theirdreamofbecomingagreatengineer.
Correct:MattandDavebelievedthattheirhardwork
intheirengineeringclasswillhelpthemrealizetheir
dreamofbecominggreatengineers.
ErrorsTested
Noun Agreement
79
VERBAL
GMATClub contributingtoeachotherslearning
PronounAntecedentDisagreement
IncorrectUseofRelativePronouns
AmbiguousandImpliedPronouns
UseofPronouns
ErrorsTested
80
VERBAL
GMAT Club 2011 87
GMATClub contributingtoeachotherslearning
ThepronounMUSTrefertoitsantecedent.
Thepronounantecedentrelationshipshouldbe
consistentthroughoutthesentence.
Incorrect: Eachofthewomenselectedforthe
scholarshipwereaskedtosubmitanapplication.
Correct: Eachofthewomenselectedforthe
scholarshipwasaskedtosubmitanapplication.
PronounErrorSubTypes
PronounAntecedentDisagreement
81
VERBAL
GMATClub contributingtoeachotherslearning
Thishappenswhenthereisadditionalinformation
betweenthepronounandantecedentmakingiteasy
tolosetrackoftherelationshipbetweenpronouns
andtheirantecedents.
Incorrect:Thelibrary,withitsmanybooksand
databases,requireaspecialmembership.
Correct: Thelibrary,withitsmanybooksand
databases,requiresaspecialmembership.
Distancebtw.pronoun&antecedent
Trap 1:
82
VERBAL
GMATClub contributingtoeachotherslearning
Theseareverygeneralpronouns many,few,
both,everyandsoon.
Incorrect:Manyofthestudentsweresurprisedto
learnthathisorherexamwasunfairlygraded.
Correct:Incorrect:Manyofthestudentswere
surprisedtolearnthattheirexamswasunfairly
graded.
Indefinite Pronouns
Trap 2:
83
VERBAL
GMATClub contributingtoeachotherslearning
AntecedentsthatSOUNDSpluralbutactualare
singularorviceversa.
Forexample,familyissingular,andpersonsis
singulartoo.Itmightbeagroupofpeopleina
family,butbeingacollectivenoun,itsstill
singular!
Trap 3:
Misleading Antecedents
84
VERBAL
GMAT Club 2011 88
GMATClub contributingtoeachotherslearning
Theyrelategroupsofwordstoanounorpronoun
which,whom,whomsoever,whereandwhy.
Twotrapsofincorrectusage:
IncorrectPronounChoiceforPeopleandThings
Differencebetweenwhoandwhom
PronounErrorSubTypes
Incorrectuseofrelativepronouns
85
VERBAL
GMATClub contributingtoeachotherslearning
WhoandWhomwillalwaysrelatetopeople.
Thedoctorthat performedthesurgerywashonored
shouldbewho
ThatandWhichwillalwaysrelatetothings,i.e.
inanimateobjects.
Themachinewho performedthesurgerywas
showcasedintheexhibit shouldbethat
Trap 1:
PronounChoice
86
VERBAL
GMATClub contributingtoeachotherslearning
Verysimpletrick!
Makethefollowingsubstitutions:
He Who
Him Whom
Thesubstitutionthatmakessensewillcorrespondwith
thecorrectwordinthatcontext.
Sometimesitmightbenecessarytorearrangethe
sentenceforittomakesense.
Trap 2:
Whovs.Whom?
87
VERBAL
GMATClub contributingtoeachotherslearning
Misplacementofadjectives(describesnouns
serene)andadverbs(describeverbs serenely)
Forexample:ThenewBMWmodelisdesignedto
drivefasterandmoreefficientthantheprevious
model.
Theusageofthewordefficientiswrongheresinceits
modifyingtheverb.Thecorrectwordusagewouldbe
efficiently
Modifiers
ErrorsTested
88
VERBAL
GMAT Club 2011 89
GMATClub contributingtoeachotherslearning
ErrorsinCountNouns(canbecounted trees,
bottles,billions).Quantifiersaremany,both,few,
several,acoupleandsoon.
ErrorsinNonCountNouns(cannotbecounted
water,liquid).Quantifiersaremuch,alittle,abit
andsoon.
Somemodifierslikeall,andsomecanbeusedfor
both.
Trap1:
89
Quantifiers
VERBAL
GMATClub contributingtoeachotherslearning
Misplacedmodifiersaremodifiersthatareplacedin
thewrongpositiononasentence.
Incorrect:Shortonmoney,thecarwasthebestoneTanya
couldfind.
Correct:Shortonmoney,Tanyaknewthatthecarwasthe
bestoneshecouldfind.
DanglingModifiersarethosewherethereferentis
completelyabsent.
Incorrect: Walkingtotheuniversity,thecathadtostop.
Correct:Walkingtotheuniversity,hehadtostopbecause
ofhiscat.
Trap2:
90
Placement
VERBAL
GMATClub contributingtoeachotherslearning
Misplacedmodifierswithrelativeclauses(like
thatorwhich)
Incorrect:Accordingtothereport,bicycleswillbe
confiscatedthathavenotbeenregisteredwiththe
university.
Correct:Accordingtothereport,bicyclesthathave
notbeenregisteredwiththeuniversitywillbe
confiscated.
Trap2:
91
Placement
VERBAL
GMATClub contributingtoeachotherslearning
Incorrectuseofcomparativedegree
IllogicalComparisons
Comparisons
ErrorsTested
92
VERBAL
GMAT Club 2011 90
GMATClub contributingtoeachotherslearning
Whentherearetwoobjectsbeingcompared,youhave
touseacomparativeword.Whenmorethantwo
objectsarecompared,youhavetouseasuperlative
word.
Incorrect:Thoughthereportersfeltthatinjuryplayedapart
inTennesseeslosstoMinnesota,Tennesseescoachsaid
thatMinnesotahadthebest teamthatnight.
Correct:Thoughthereportersfeltthatinjuryplayedapart
inTennesseeslosstoMinnesota,Tennesseescoachsaid
thatMinnesotahadthebetterteamthatnight.
ComparisonSubType
Degree ofComparison
93
VERBAL
GMATClub contributingtoeachotherslearning
Onlysimilarobjectscanbecompared.Youcannot
compareahumantoadog.
Incorrect: ThoughtherecentLaysfoodpoisoning
incidenthascausedsomebuyerstoquestionitsquality,
theLayschipsaresoldmoreoftenthananyotherfood
company.
Correct: ThoughtherecentLaysfoodpoisoningincident
hascausedsomebuyerstoquestionitsquality,theLays
chipsaresoldmoreoftenthanthoseproducedbyany
otherfoodcompany.
ComparisonSubType
Illogical Comparison
94
VERBAL
GMATClub contributingtoeachotherslearning
ParallelVerbsandVerbFormErrors
ParallelNounErrors
ParallelPrepositionsandArticles
ParallelConjunctions
ParallelComparisons
Parallelism
ErrorsTested
95
VERBAL
GMATClub contributingtoeachotherslearning
Nounsarenaturallyparallel.Butwhentheyarein
theformwithaningattheend(gerunds),theyare
confusedforverbs.
Incorrect: Thecommonsymptomsofthestomachflu
areindigestion,vomitinganddrinkinglesswaterthan
usual.
Correct: Thecommonsymptomsofthestomachflue
areindigestion,vomitinganddecreasedwaterintake.
ParallelismSubType
VerbErrors
96
VERBAL
GMAT Club 2011 91
GMATClub contributingtoeachotherslearning
Whenasentencehastwoormoresimilarparts
linkedbyaconjunction,eachparthastobeofthe
sameverbform.
Incorrect: Thenewexampatternrequiresstudentsto
entertheirstudentIDsandsigningtheirnamesonthe
roster.
Correct: Thenewexampatternrequiresstudentsto
entertheirstudentIDsandsigntheirnamesonthe
roster.
NounErrors
ParallelismSubType
97
VERBAL
GMATClub contributingtoeachotherslearning
Aprepositionandanarticlemusteitherbeusedby
allpartsofasentenceorbyjustthefirstpart.
Incorrect: Bydoingeachassignment,turningin
homeworksontimeandby attendingalltheclasses,
onemightbeabletoobtainthehighestgradeinthe
class.
Correct: Bydoingeachassignment,turningin
homeworksontime,andattendingalltheclasses,one
mightbeabletoobtainthehighestgradeintheclass.
Prepositional/Article Errors
ParallelismSubType
98
VERBAL
GMATClub contributingtoeachotherslearning
Theconjunctionsusedmustbeparallelinstructure.
Someexamples:
Incorrect: Therestaurantnotonlyhiredanewchef,butit
also expandeditsseatingcapacity.
Correct: Therestaurantnotonlyhiredanewchef,butalso
expandeditsseatingcapacity.
ParallelConjunctions
Neither nor Eitheror
Notonly butalso Bothand
Whether or Asas
ParallelismSubType
99
VERBAL
GMATClub contributingtoeachotherslearning
ParallelComparisons
Whencomparisons areleftunparallel,thatcan
amount toanerroraswell.
Incorrect: In thebook,theauthormentionsthathe
foundeditingpicturesmuchmorearduousand
difficultthanwhenhehadtotaketheactualpictures.
Correct: Inthebook,theauthormentionsthathe
foundeditingpictures muchmorearduousand
difficultthantakingpictures.
ParallelismSubType
100
VERBAL
GMAT Club 2011 92
GMATClub contributingtoeachotherslearning
Usageoffragmentsordependentclauseson
eithersideofasemicolon
Incorrect:Theguineapigisnotatruepig;rather,a
rodentthatbelongstotheCaviidae family.
Correct: Theguineapigisnotatruepig,butrathera
rodentthatbelongstotheCaviidae family.
Correct: Theguineapigisnotatruepig;itsarodent
thatbelongstotheCaviidae family.
SemiColons
ErrorsTested
101
VERBAL
GMATClub contributingtoeachotherslearning
TheGMATwillpreferusageofthemostconcise
sentencespossible.
Incorrect: Hedecidedtonotpurchasethecarduetothe
factthat itwastooexpensive.
Correct: Hedecidedtonotpurchasethecarbecauseit
wastooexpensive.
Incorrect: Inadditiontoplayingtheguitar,Robalso plays
theviola.
Correct: Inadditiontoplayingtheguitar,Robplaysthe
viola.
WordinessandRedundancy
ErrorsTested
102
VERBAL
GMATClub contributingtoeachotherslearning
Very,verycommonerrortested.
Aphrasethatiscommonlyacceptedascorrect
eventhoughitsgrammaticallyinconsistent
Anidiomerrorwillmisrepresenttheidiom.
Idiomsareprovidedwithrightandwrongusage
inthecardsthatfollow.
Idioms
ErrorsTested
103
VERBAL
GMATClub contributingtoeachotherslearning
Amongvs.Between
Among:
Used when more than two items are in question.
Example: He was the best among three candidates.
Between:
Used when two items are in question
Example: He was the best between the two of them.
ErrorsTestedIdiomRules
104
VERBAL
GMAT Club 2011 93
GMATClub contributingtoeachotherslearning
Bothvs.Each
Both:
Used to point out similarities.
Example: Both of them were good at swimming.
Each:
Used to point out differences/dissimilarities. (Always
singular)
Example: Each girl had her own niche.
ErrorsTestedIdiomRules
105
VERBAL
GMATClub contributingtoeachotherslearning
Twicevs.Double
Twice:
Twice/Thrice etc. are used for comparison
Example: The Toyota was twice as fast as the Honda.
Double:
Used as a verb only.
Example: He more than doubled his wealth by
investing in stocks.
ErrorsTestedIdiomRules
106
VERBAL
GMATClub contributingtoeachotherslearning
EachOthervs.OneAnother
Each Other:
Used to compare two things.
Example: They loved each other dearly.
One another:
Used to compare more than two things.
Example: The three brothers loved one another dearly.
ErrorsTestedIdiomRules
107
VERBAL
GMATClub contributingtoeachotherslearning
Ifvs.Whether
If:
Primarily used only in If else sentences.
Example: If this is true, the market will collapse. Else, it will
be okay tomorrow.
Whether:
Used more frequently as a comparison.
Example: Whether or not he chooses to accept the prize is
up to him.
ErrorsTestedIdiomRules
108
VERBAL
GMAT Club 2011 94
GMATClub contributingtoeachotherslearning
Likevs.SuchAs
Like:
Used when indicating similarities between things.
Example: Like John, Amy was a violinist too.
Such as:
Used to list examples.
Example: There are different kinds of tigers such as the
Bengal Tigers, white tigers and so on.
ErrorsTestedIdiomRules
109
VERBAL
GMATClub contributingtoeachotherslearning
AbilityTo
Correct:Dolphinshavetheabilitytoemitlowfrequency
whistles
Incorrect:Dolphinshavetheabilityofemittinglow
frequencywhistles.
Actas/like
Correct:Asignaturecanactasa legalattetato.
Incorrect:Asignaturecanactlikealegalattetato.
Correct:Hewasactinglikeachild.
Incorrect:Hewasactingasachild.
ErrorsTestedIdioms
110
VERBAL
GMATClub contributingtoeachotherslearning
Agreeupon/to
Correct: Weagreeduponthedateofthemeeting
Incorrect:Weagreedtothedateofthemeeting.
Correct:Weagreedtoshareourroom.
Incorrect:Weagreeduponsharingourroom.
Allowfor
Correct: Onemustalwaysallowforamarginoferror
whilecalculatingaxialloads.
Incorrect:Onemustalwaysallowtohaveamarginof
errorwhilecalculatingaxialloads.
ErrorsTestedIdioms
111
VERBAL
GMATClub contributingtoeachotherslearning
Appealto
Correct:Iappealedtohissenseofjustice.
Incorrect:Iappealedforhissenseofjustice.
Areindangerof
Correct:Thegiantpandabearsareindangerof
extinction.
Incorrect:Thegiantpandabearshaveadangerof
dyingfromdeforestation.
ErrorsTestedIdioms
112
VERBAL
GMAT Club 2011 95
GMATClub contributingtoeachotherslearning
Asanadolescent
Correct: Asanadolescent,hesufferedfromattention
disorder
Incorrect:Whileinadolescence,hesufferedfrom
attentiondisorder.
Asgoodas
Correct:Itsasgoodasnew.
ErrorsTestedIdioms
113
VERBAL
GMATClub contributingtoeachotherslearning
Associatewith
Correct:Iassociatespringwithflowers.
Incorrect:IalwaysassociatewatertoNiagarafalls.
Attendto
Correct:Ihavetoattendtosomeduties
Incorrect:Ihavetoattendforsomeduties.
ErrorsTestedIdioms
114
VERBAL
GMATClub contributingtoeachotherslearning
Attributeto
Correct:Iattributemysuccesstohardwork.
Incorrect:Iattributemysuccesswithhardwork.
Baseon
Correct:Thedecisionwasmadebasedonmultiple
criteria.
Incorrect:Thedecisionwasmadebasedofmultiple
criteria.
ErrorsTestedIdioms
115
VERBAL
GMATClub contributingtoeachotherslearning
Begintoseedaylight
Correct:Afterworkingontheprojectallnight,Iam
finallybeginningtoseedaylight.
Incorrect:Afterworkingontheprojectallnight,Iam
finallybeginningtoviewthedaylight.
Between<>and<>
Correct:Ihadtochoosebetweenchocolatesand
cakes.
Incorrect:Ihadtochoosebetweenchocolateswith
cakes.
ErrorsTestedIdioms
116
VERBAL
GMAT Club 2011 96
GMATClub contributingtoeachotherslearning
Carefor/about
Correct:Idontcaremuchforsweets
Correct:Icareaboutheralot.
Claimto/Claimthat
Correct:Ineverclaimedtopossessremarkablesinging
talent.
Incorrect:Ineverclaimedthatpossessremarkable
singingtalent.
Correct:IneverclaimedthatIpossessedremarkable
talent.
ErrorsTestedIdioms
117
VERBAL
GMATClub contributingtoeachotherslearning
Cometoadeadend
Correct:Hecametoadeadendafterresearchingtopics.
Incorrect: Hecamebyadeadendafterresearchingtopics.
Compareto/with
Correct:Inevercomparedmyselftoher.
Incorrect:Inevercomparedmyselfforher.
Correct:Letscomparetheprosofthesituationwiththe
cons.
Incorrect:Letscomparetheprosofthesituationforthe
cons.
ErrorsTestedIdioms
118
VERBAL
GMATClub contributingtoeachotherslearning
Consider(as)
Correct:Idontconsiderhimaseriouscontender
Incorrect:Idontconsiderhimwithaserious
contender
Conformto
Correct:Youmustconformtothestandards
Incorrect:Youmustconformwiththestandards
ErrorsTestedIdioms
119
VERBAL
GMATClub contributingtoeachotherslearning
Contrastto/with
Correct:Incontrasttothepreviousreport,thisoneisbetter
Incorrect:Incontrastwiththepreviousreport,thisoneis
better.
Correct:Theredbackgroundcontrastsnicelywiththeblue
flowers
Incorrect:Theredbackgroundcontrastsnicelytotheblue
flowers.
Counton
Correct:IknewthatIcouldcountonyou.
Incorrect:IknewthatIcouldcountforyou.
ErrorsTestedIdioms
120
VERBAL
GMAT Club 2011 97
GMATClub contributingtoeachotherslearning
Creditfor/to/with
Correct:Youshouldtakecreditforwhatyoudid.
Incorrect:Youshouldtakecredittowhatyoudid.
Correct:Icreditmysuccesstomyhardwork.
Incorrect:Icreditmysuccessformyhardwork.
Correct:Heiscreditedwiththediscoveryofpenicillin.
Incorrect:Heiscreditedfor/tothediscoveryofpenicillin.
ErrorsTestedIdioms
121
VERBAL
GMATClub contributingtoeachotherslearning
Debateabout
Correct:Therewasaragingdebateaboutstandards.
Incorrect:Therewasaragingdebateforstandards.
Decideon
Correct:Iamyettodecideonacolorforthewall.
Incorrect:Iamyettodecideaboutacolorforthe
wall.
ErrorsTestedIdioms
122
VERBAL
GMATClub contributingtoeachotherslearning
Declared<>
Correct:Theteacherdeclaredallexamsworthy.
Incorrect:Theteacherdeclaredallexamsasworthy.
Defineas
Correct:Lightisdefinedasradiation.
Incorrect:Lightisdefinedforradiation.
ErrorsTestedIdioms
123
VERBAL
GMATClub contributingtoeachotherslearning
Delightedto
Correct:Iamdelightedtoacceptthisprize.
Incorrect:Iamdelightedforacceptingthisprize.
Differentfrom
Correct:Thissandwichisdifferentfromtheothers.
Incorrect:Thissandwichisdifferentwiththeothers.
ErrorsTestedIdioms
124
VERBAL
GMAT Club 2011 98
GMATClub contributingtoeachotherslearning
Distinguishbetween/from
Correct:Ihadtodistinguishbetweengoodandbad.
Incorrect:Ihadtodistinguishfromgoodandbad.
Correct:Ihadtodistinguishgoodfrombad.
Incorrect:Ihadtodistinguishgoodwithbad.
Drawaline
Correct:Wehadtodrawalinesomewhere.
Incorrect:Wehadtodrawthelinebefore.
ErrorsTestedIdioms
125
VERBAL
GMATClub contributingtoeachotherslearning
Drawattentionto
Correct:Ihatetodrawattentiontothefactthatthe
qualityoffoodhasgonedown.
Incorrect:Ihatetodrawattentionforthefactthatthe
qualityoffoodhasgonedown.
Drawupon
Correct:Wehadtodrawuponthereservestokeepthe
carrunning.
Incorrect:Wehadtodrawonthereservestokeepthe
carrunning.
ErrorsTestedIdioms
126
VERBAL
GMATClub contributingtoeachotherslearning
Drawto
Correct:Hewasdrawntoherfromthemomenthe
mether.
Incorrect:Hewasdrawnforherfromthemomenthe
mether.
Easiersaidthandone
Correct:Itsalwayseasiersaidthandone.
Incorrect:Itsalwayseasiersaidthantobedone.
ErrorsTestedIdioms
127
VERBAL
GMATClub contributingtoeachotherslearning
Electas/to
Correct:Hewaselectedtooffice.
Incorrect:Hewaselectedtoofficer.
Correct:Hewaselectedasanofficer.
Incorrect:Hewaselectedasoffice.
Easiersaidthandone
Correct:Itsalwayseasiersaidthandone.
Incorrect:Itsalwayseasiersaidthantobedone.
ErrorsTestedIdioms
128
VERBAL
GMAT Club 2011 99
GMATClub contributingtoeachotherslearning
Indicatethat
Correct:Studiesindicatethatstressisacommoncause
ofheartattacks.
Incorrect:Studiesindicateaboutstressbeingacommon
causeofheartattacks.
Inorderto
Correct:Shebeganstudyinginordertogetagoodgrade
intheclass.
Incorrect:Shebeganstudyinginorderthatshegota
goodgradeintheclass.
ErrorsTestedIdioms
129
VERBAL
GMATClub contributingtoeachotherslearning
Justas<>,so<>
Correct:JustasTyrawasconsideredforthescholarship,so
wasMia.
Incorrect:JustasTyrawasconsideredforthescholarship,
Miaasalsoconsidered.
Knownto
Correct:Evenasastudent,Rickwasknowntodothings
differently.
Incorrect:Evenasastudent,Rickwasknownaswantingto
dothingsdifferently.
ErrorsTestedIdioms
130
VERBAL
GMATClub contributingtoeachotherslearning
Left,rightandcenter
Correct:Shewasshootingemailstopeopleleft,rightand
center.
Incorrect:Shewas,leftrightandcenter,sendingemails.
Knownto
Correct:Evenasastudent,Rickwasknowntodothings
differently.
Incorrect:Evenasastudent,Rickwasknownaswantingto
dothingsdifferently.
ErrorsTestedIdioms
131
VERBAL
GMATClub contributingtoeachotherslearning
Ameansto
Correct:Sheonlyvieweditasameanstotheend.
Incorrect:Forsomepeople,moneyisconsidereda
meansfor/ofanend.
Mistakenfor
Correct:Thetwinswereoftenmistakenforone
another.
Incorrect:Thetwinswereoftenmistakenasone
another.
ErrorsTestedIdioms
132
VERBAL
GMAT Club 2011 100
GMATClub contributingtoeachotherslearning
Morethanever
Correct:IregretnotgoingtoEurope,nowmorethanever.
Incorrect:IregretnotgoingtoEurope,nowmorethan
never.
Nativeof/to
Correct:SheisanativeofVienna.
Incorrect:SheisanativetoVienna.
Correct:TheBengalTigerisnativetoIndia.
Incorrect:TheBengalTigerisnativeofIndia.
ErrorsTestedIdioms
133
VERBAL
GMATClub contributingtoeachotherslearning
Prohibitfrom
Correct:Onlywhenweprohibitpeoplefrombuyingdrinks
forminors,willwereducetheproblemofunderage
drinking.
Incorrect:Onlywhenweprohibitpeopletobuydrinksfor
minors,willwereducetheproblemofunderagedrinking.
Rangefrom
Correct:Thequalityoftheseproductsrangefromgoodto
excellent.
Incorrect:Thequalityoftheseproductsrangebetween
goodtoexcellent.
ErrorsTestedIdioms
134
VERBAL
GMATClub contributingtoeachotherslearning
Reluctantto
Correct:Shewasreluctanttotakeonsuchahugetask.
Incorrect:Shewasreluctantabouttakingonsucha
hugetask.
Requireof
Correct:Itisrequiredofallcandidatestoreporthere
daily.
Incorrect:Itisrequiredfromallcandidatestoreport
heredaily.
ErrorsTestedIdioms
135
VERBAL
GMATClub contributingtoeachotherslearning
Seemto
Correct:Heseemedtobehidingsomethinginhispocket.
Incorrect:Heseemedashewashidingsomethinginhis
pocket.
Takeadvantageof
Correct:Hetookadvantageofalltheopportunitieshehad.
Incorrect:Hetookadvantageforalltheopportunitieshe
had.
ErrorsTestedIdioms
136
VERBAL
GMAT Club 2011 101
GMATClub contributingtoeachotherslearning
Idioms
Whichsentenceisidiomaticallycorrect?
A. WillFerrell'scameoscenesaresofunnyas
anythinghe'severdone.
B. WillFerrell'scameoscenesareasfunnythat
anythinghe'severdone.
C. WillFerrell'scameoscenesareasfunnyas
anythinghe'severdone.
137
VERBAL
GMATClub contributingtoeachotherslearning
Idioms
Correctanswer:
A. WillFerrell'scameoscenesaresofunnyas
anythinghe'severdone.
B. WillFerrell'scameoscenesareasfunnythat
anythinghe'severdone.
C. WillFerrell'scameoscenesareasfunnyas
anythinghe'severdone.
138
VERBAL
GMATClub contributingtoeachotherslearning
Idioms
Whichsentenceisidiomaticallycorrect?
A. Themarkethasconfidencethat thefirmwilladd
back leveragetoincreaseprofits.
B. Themarkethasconfidencein thefirm'sabilityto
addback leveragetoincreaseprofits.
C. Themarkethasconfidencein thefirmtoadd
back leveragetoincreaseprofits.
139
VERBAL
GMATClub contributingtoeachotherslearning
Idioms
Correctanswer:
A. Themarkethasconfidencethat thefirmwilladd
back leveragetoincreaseprofits.
B. Themarkethasconfidencein thefirm'sabilityto
addback leveragetoincreaseprofits.
C. Themarkethasconfidencein thefirmtoadd
back leveragetoincreaseprofits.
140
VERBAL
GMAT Club 2011 102
GMATClub contributingtoeachotherslearning
Idioms
Whichsentenceisidiomaticallycorrect?
A. Whatthesescorestellusisthatrigorislackingin
someschools.
B. Whatthesescorestellusisthatsomeschools
lackof rigor.
C. Whatthesescorestellusisthatthere'salackof
rigorinsomeschools.
141
VERBAL
GMATClub contributingtoeachotherslearning
Idioms
Correctanswer:
A. Whatthesescorestellusisthatrigorislackingin
someschools.
B. Whatthesescorestellusisthatsomeschools
lackof rigor.
C. Whatthesescorestellusisthatthere'salackof
rigorinsomeschools.
142
VERBAL
GMATClub contributingtoeachotherslearning
Idioms
Whichsentenceisidiomaticallycorrect?
A. Thecityindustrialistshavedemanded thestate
governmenttomeet thepromisesmadebychief
ministerAshokChavan.
B. Thecityindustrialistshavedemandedthat the
stategovernmentmeet thepromisesmadeby
chiefministerAshokChavan.
C. Thecityindustrialistshavedemandedthat the
stategovernmentmet thepromisesmadeby
chiefministerAshokChavan.
143
VERBAL
GMATClub contributingtoeachotherslearning
Idioms
Correctanswer:
A. Thecityindustrialistshavedemanded thestate
governmenttomeet thepromisesmadebychief
ministerAshokChavan.
B. Thecityindustrialistshavedemandedthat the
stategovernmentmeet thepromisesmadeby
chiefministerAshokChavan.
C. Thecityindustrialistshavedemandedthat the
stategovernmentmet thepromisesmadeby
chiefministerAshokChavan.
144
VERBAL
GMAT Club 2011 103
GMATClub contributingtoeachotherslearning
Idioms
Whichsentenceisidiomaticallycorrect?
A. Tensionshaveflaredinsomepartsofthecountry
between blackswith Hispanics.
B. Tensionshaveflaredinsomepartsofthecountry
between blacksand Hispanics.
C. Tensionshaveflaredinsomepartsofthecountry
among blacksand Hispanics.
145
VERBAL
GMATClub contributingtoeachotherslearning
Idioms
Correctanswer:
A. Tensionshaveflaredinsomepartsofthecountry
between blackswith Hispanics.
B. Tensionshaveflaredinsomepartsofthecountry
between blacksand Hispanics.
C. Tensionshaveflaredinsomepartsofthecountry
among blacksand Hispanics.
146
VERBAL
GMATClub contributingtoeachotherslearning
Idioms
Whichsentenceisidiomaticallycorrect?
A. TheproblemisDisney'sabilitytotake controlof
someofthebestknowncharactersquicklyisvery
limited.
B. TheproblemisDisney'sabilityoftaking control
ofsomeofthebestknowncharactersquicklyis
verylimited.
C. TheproblemisDisney'sabilityfortaking control
ofsomeofthebestknowncharactersquicklyis
verylimited.
147
VERBAL
GMATClub contributingtoeachotherslearning
Idioms
Correctanswer:
A. TheproblemisDisney'sabilitytotake controlof
someofthebestknowncharactersquicklyisvery
limited.
B. TheproblemisDisney'sabilityoftaking control
ofsomeofthebestknowncharactersquicklyis
verylimited.
C. TheproblemisDisney'sabilityfortaking control
ofsomeofthebestknowncharactersquicklyis
verylimited.
148
VERBAL
GMAT Club 2011 104
GMATClub contributingtoeachotherslearning
Idioms
Whichsentenceisidiomaticallycorrect?
A. Areadingabove50percentindicates the
manufacturingeconomytobe generally
expanding.
B. Areadingabove50percentindicatesthat the
manufacturingeconomyis generallyexpanding.
C. Areadingabove50percentindicates the
manufacturingeconomyis generallyexpanding.
149
VERBAL
GMATClub contributingtoeachotherslearning
Idioms
Correctanswer:
A. Areadingabove50percentindicates the
manufacturingeconomytobe generally
expanding.
B. Areadingabove50percentindicatesthat the
manufacturingeconomyis generallyexpanding.
C. Areadingabove50percentindicates the
manufacturingeconomyis generallyexpanding.
150
VERBAL
GMATClub contributingtoeachotherslearning
Idioms
Whichsentenceisidiomaticallycorrect?
A. Incontrastto thefirsttwooptionscentralizing
decisionmaking,aCapandTradesystemwillnot.
B. Ascontrastedwith thefirsttwooptions,aCap
andTradesystemwilldecentralize decision
making.
C. Incontrastto thefirsttwooptions,aCapand
Tradesystemwilldecentralize decisionmaking.
151
VERBAL
GMATClub contributingtoeachotherslearning
Idioms
Correctanswer:
A. Incontrastto thefirsttwooptionscentralizing
decisionmaking,aCapandTradesystemwillnot.
B. Ascontrastedwith thefirsttwooptions,aCap
andTradesystemwilldecentralize decision
making.
C. Incontrastto thefirsttwooptions,aCapand
Tradesystemwilldecentralize decisionmaking.
152
VERBAL
GMAT Club 2011 105
GMATClub contributingtoeachotherslearning
Idioms
Whichsentenceisidiomaticallycorrect?
A. Thisreporthasbeenissuedbytheassociation
since1931,excepting afouryearinterruption
duringWorldWarII.
B. Thisreporthasbeenissuedbytheassociation
since1931,exceptfor afouryearinterruption
duringWorldWarII.
C. Thisreporthasbeenissuedbytheassociation
since1931,withtheexceptionof afouryear
interruptionduringWorldWarII.
153
VERBAL
GMATClub contributingtoeachotherslearning
Idioms
Correctanswer:
A. Thisreporthasbeenissuedbytheassociation
since1931,excepting afouryearinterruption
duringWorldWarII.
B. Thisreporthasbeenissuedbytheassociation
since1931,exceptfor afouryearinterruption
duringWorldWarII.
C. Thisreporthasbeenissuedbytheassociation
since1931,withtheexceptionof afouryear
interruptionduringWorldWarII.
154
VERBAL
GMATClub contributingtoeachotherslearning
Idioms
Whichsentenceisidiomaticallycorrect?
A. Itsgrossmarginrosefrom 22%upto 25%,butits
operatingmarginfellfrom 7%downto 4%.
B. Itsgrossmarginrosefrom 22%to 25%,butits
operatingmarginfellfrom 7%to 4%.
C. Itsgrossmarginrosefrom 22%until 25%,butits
operatingmarginfellfrom 7%till 4%.
155
VERBAL
GMATClub contributingtoeachotherslearning
Idioms
Correctanswer:
A. Itsgrossmarginrosefrom 22%upto 25%,butits
operatingmarginfellfrom 7%downto 4%.
B. Itsgrossmarginrosefrom 22%to 25%,butits
operatingmarginfellfrom 7%to 4%.
C. Itsgrossmarginrosefrom 22%until 25%,butits
operatingmarginfellfrom 7%till 4%.
156
VERBAL
GMAT Club 2011 106
GMATClub contributingtoeachotherslearning
Idioms
Whichsentenceisidiomaticallycorrect?
A. "Ibelievethatwealthisameansfor anend,not
anendinitself".
B. "Ibelievethatwealthisameansof anend,not
anendinitself".
C. "Ibelievethatwealthisameansto anend,not
anendinitself".
157
VERBAL
GMATClub contributingtoeachotherslearning
Idioms
Correctanswer:
A. "Ibelievethatwealthisameansfor anend,not
anendinitself".
B. "Ibelievethatwealthisameansof anend,not
anendinitself".
C. "Ibelievethatwealthisameansto anend,not
anendinitself".
158
VERBAL
GMATClub contributingtoeachotherslearning
Idioms
Whichsentenceisidiomaticallycorrect?
A. TheancientChineseTaoistsdistinguished
intercourseand orgasm.
B. TheancientChineseTaoistsdistinguished
intercoursefrom orgasm.
C. TheancientChineseTaoistsdistinguished
between intercourseand orgasm.
159
VERBAL
GMATClub contributingtoeachotherslearning
Idioms
Correctanswer:
A. TheancientChineseTaoistsdistinguished
intercourseand orgasm.
B. TheancientChineseTaoistsdistinguished
intercoursefrom orgasm.
C. TheancientChineseTaoistsdistinguished
between intercourseand orgasm.
160
VERBAL
GMAT Club 2011 107
GMATClub contributingtoeachotherslearning
Idioms
Whichsentenceisidiomaticallycorrect?
A. Itwouldhavebeeneasyenoughfor hertobuy a
ranchstylehousethatshecouldeasilyandlogically
decoratewithScandinavianstylefurniture.
B. Itwouldhavebeeneasyenoughsothat shebought
aranchstylehousethatshecouldeasilyandlogically
decoratewithScandinavianstylefurniture.
C. Itwouldhavebeeneasyenoughastobuy aranch
stylehousethatshecouldeasilyandlogically
decoratewithScandinavianstylefurniture.
161
VERBAL
GMATClub contributingtoeachotherslearning
Idioms
Correctanswer:
A. Itwouldhavebeeneasyenoughfor hertobuy a
ranchstylehousethatshecouldeasilyandlogically
decoratewithScandinavianstylefurniture.
B. Itwouldhavebeeneasyenoughsothat shebought
aranchstylehousethatshecouldeasilyandlogically
decoratewithScandinavianstylefurniture.
C. Itwouldhavebeeneasyenoughastobuy aranch
stylehousethatshecouldeasilyandlogically
decoratewithScandinavianstylefurniture.
162
VERBAL
GMATClub contributingtoeachotherslearning
Idioms
Whichsentenceisidiomaticallycorrect?
A. iPhoneusersdownloadtwotofourtimesas
manymore games,video,andotherWebdata
than othersmartphoneusers.
B. iPhoneusersdownloadtwotofourtimesas
manymore games,video,andotherWebdataas
othersmartphoneusers.
C. iPhoneusersdownloadtwotofourtimesas
many games,video,andotherWebdataas other
smartphoneusers.
163
VERBAL
GMATClub contributingtoeachotherslearning
Idioms
Correctanswer:
A. iPhoneusersdownloadtwotofourtimesas
manymore games,video,andotherWebdata
than othersmartphoneusers.
B. iPhoneusersdownloadtwotofourtimesas
manymore games,video,andotherWebdataas
othersmartphoneusers.
C. iPhoneusersdownloadtwotofourtimesas
many games,video,andotherWebdataas other
smartphoneusers.
164
VERBAL
GMAT Club 2011 108
GMATClub contributingtoeachotherslearning
Idioms
Whichsentenceisidiomaticallycorrect?
A. Wehaveonechanceinamillionof winningthe
WorldCup.
B. Wehaveoneinamillionchancesto winthe
WorldCup.
C. Wehaveonechanceinamillionfor winningthe
WorldCup.
165
VERBAL
GMATClub contributingtoeachotherslearning
Idioms
Correctanswer:
A. Wehaveonechanceinamillionof winningthe
WorldCup.
B. Wehaveoneinamillionchancesto winthe
WorldCup.
C. Wehaveonechanceinamillionfor winningthe
WorldCup.
166
VERBAL
GMATClub contributingtoeachotherslearning
Idioms
Whichsentenceisidiomaticallycorrect?
A. Oureconomicwellbeingisgenerallydetermined
from theamountofgoodsandservicesthatwe
consume.
B. Oureconomicwellbeingisgenerallydetermined
by theamountofgoodsandservicesthatwe
consume.
C. Oureconomicwellbeingisgenerallydetermined
through theamountofgoodsandservicesthat
weconsume.
167
VERBAL
GMATClub contributingtoeachotherslearning
Idioms
Correctanswer:
A. Oureconomicwellbeingisgenerallydetermined
from theamountofgoodsandservicesthatwe
consume.
B. Oureconomicwellbeingisgenerallydetermined
by theamountofgoodsandservicesthatwe
consume.
C. Oureconomicwellbeingisgenerallydetermined
through theamountofgoodsandservicesthat
weconsume.
168
VERBAL
GMAT Club 2011 109
GMATClub contributingtoeachotherslearning
Idioms
Whichsentenceisidiomaticallycorrect?
A. Sheiscreditedfor solvingnumerouscases.
B. Sheiscreditedwith solvingnumerouscases.
C. Sheiscreditedasbeing agreatdetective.
169
VERBAL
GMATClub contributingtoeachotherslearning
Idioms
Correctanswer:
A. Sheiscreditedfor solvingnumerouscases.
B. Sheiscreditedwith solvingnumerouscases.
C. Sheiscreditedasbeing agreatdetective.
170
VERBAL
GMATClub contributingtoeachotherslearning
Idioms
Whichsentenceisidiomaticallycorrect?
A. Whyhasn'tCongressmandatedthat thetotal
premiumcostbe shownaswagesoneverypay
stubbutnottaxed?
B. Whyhasn'tCongressamandatefor thetotal
premiumcosttobe shownaswagesoneverypay
stubbutnottaxed?
C. Whyhasn'tCongressmandatedthat thetotal
premiumcostare shownaswagesoneverypay
stubbutnottaxed?
171
VERBAL
GMATClub contributingtoeachotherslearning
Idioms
Correctanswer:
A. Whyhasn'tCongressmandatedthat thetotal
premiumcostbe shownaswagesoneverypay
stubbutnottaxed?
B. Whyhasn'tCongressamandatefor thetotal
premiumcosttobe shownaswagesoneverypay
stubbutnottaxed?
C. Whyhasn'tCongressmandatedthat thetotal
premiumcostare shownaswagesoneverypay
stubbutnottaxed?
172
VERBAL
GMAT Club 2011 110
GMATClub contributingtoeachotherslearning
Idioms
Whichsentenceisidiomaticallycorrect?
A. A13yearoldgirlwasmistakenlyabductedinMarch
2008bytraffickerswhowerebelievedtohave
mistaken heras thenieceofasuspecteddrugdealer.
B. A13yearoldgirlwasmistakenlyabductedinMarch
2008bytraffickerswhowerebelievedtohave
mistaken herfor thenieceofasuspecteddrug
dealer.
C. A13yearoldgirlwasmistakenlyabductedinMarch
2008bytraffickerswhowerebelievedtohave
mistaken herto thenieceofasuspecteddrugdealer.
173
VERBAL
GMATClub contributingtoeachotherslearning
Idioms
Correctanswer:
A. A13yearoldgirlwasmistakenlyabductedinMarch
2008bytraffickerswhowerebelievedtohave
mistaken heras thenieceofasuspecteddrugdealer.
B. A13yearoldgirlwasmistakenlyabductedinMarch
2008bytraffickerswhowerebelievedtohave
mistaken herfor thenieceofasuspecteddrug
dealer.
C. A13yearoldgirlwasmistakenlyabductedinMarch
2008bytraffickerswhowerebelievedtohave
mistaken herto thenieceofasuspecteddrugdealer.
174
VERBAL
GMATClub contributingtoeachotherslearning
Idioms
Whichsentenceisidiomaticallycorrect?
A. Butbothconsider theproblemsas necessary
inconveniencestofeaturetheirconferencesand
programs.
B. Butbothconsider theproblemstobe necessary
inconveniencestofeaturetheirconferencesand
programs.
C. Butbothconsider theproblemsnecessary
inconveniencestofeaturetheirconferencesand
programs.
175
VERBAL
GMATClub contributingtoeachotherslearning
Idioms
Correctanswer:
A. Butbothconsider theproblemsas necessary
inconveniencestofeaturetheirconferencesand
programs.
B. Butbothconsider theproblemstobe necessary
inconveniencestofeaturetheirconferencesand
programs.
C. Butbothconsider theproblemsnecessary
inconveniencestofeaturetheirconferencesand
programs.
176
VERBAL
GMAT Club 2011 111
GMATClub contributingtoeachotherslearning
Idioms
Whichsentenceisidiomaticallycorrect?
A. Thedrugmakerproposed thelitigationbe
centralizedintheUSDistrictCourtforthe
NorthernDistrictofOhio.
B. Thedrugmakerproposedthat thelitigationisto
be centralizedintheUSDistrictCourtforthe
NorthernDistrictofOhio.
C. Thedrugmakerproposedthat thelitigationbe
centralizedintheUSDistrictCourtforthe
NorthernDistrictofOhio.
177
VERBAL
GMATClub contributingtoeachotherslearning
Idioms
Correctanswer:
A. Thedrugmakerproposed thelitigationbe
centralizedintheUSDistrictCourtforthe
NorthernDistrictofOhio.
B. Thedrugmakerproposedthat thelitigationisto
be centralizedintheUSDistrictCourtforthe
NorthernDistrictofOhio.
C. Thedrugmakerproposedthat thelitigationbe
centralizedintheUSDistrictCourtforthe
NorthernDistrictofOhio.
178
VERBAL
GMATClub contributingtoeachotherslearning
Idioms
Whichsentenceisidiomaticallycorrect?
A. Theastronautswouldprobablybestusetheir
remainingtimelivingandworkingonMars
insteadof dyingathome.
B. Theastronautswouldprobablybestusetheir
remainingtimelivingandworkingonMarsrather
dyingathome.
C. Theastronautswouldprobablybestusetheir
remainingtimelivingandworkingonMarsrather
than dyingathome.
179
VERBAL
GMATClub contributingtoeachotherslearning
Idioms
Correctanswer:
A. Theastronautswouldprobablybestusetheir
remainingtimelivingandworkingonMars
insteadof dyingathome.
B. Theastronautswouldprobablybestusetheir
remainingtimelivingandworkingonMarsrather
dyingathome.
C. Theastronautswouldprobablybestusetheir
remainingtimelivingandworkingonMarsrather
than dyingathome.
180
VERBAL
GMAT Club 2011 112
GMATClub contributingtoeachotherslearning
Idioms
Whichsentenceisidiomaticallycorrect?
A. Thecompanyrequires himhold stockvaluedat
$4.4million,fourtimeshisannualbasesalaryof
$1.1million.
B. Thecompanyrequires himtohold stockvalued
at$4.4million,fourtimeshisannualbasesalary
of$1.1million.
C. Thecompanyrequires himholding stockvalued
at$4.4million,fourtimeshisannualbasesalary
of$1.1million.
181
VERBAL
GMATClub contributingtoeachotherslearning
Idioms
Correctanswer:
A. Thecompanyrequires himhold stockvaluedat
$4.4million,fourtimeshisannualbasesalaryof
$1.1million.
B. Thecompanyrequires himtohold stockvalued
at$4.4million,fourtimeshisannualbasesalary
of$1.1million.
C. Thecompanyrequires himholding stockvalued
at$4.4million,fourtimeshisannualbasesalary
of$1.1million.
182
VERBAL
GMATClub contributingtoeachotherslearning
Idioms
Whichsentenceisidiomaticallycorrect?
A. Cubanpovertyisaresultof theAmericantrade
embargo.
B. Theresultof theAmericanembargowas Cuba
impoverished.
C. Resultingfrom theAmericanembargo,Cuba
impoverished.
183
VERBAL
GMATClub contributingtoeachotherslearning
Idioms
Correctanswer:
A. Cubanpovertyisaresultof theAmericantrade
embargo.
B. Theresultof theAmericanembargowas Cuba
impoverished.
C. Resultingfrom theAmericanembargo,Cuba
impoverished.
184
VERBAL
GMAT Club 2011 113
GMATClub contributingtoeachotherslearning
Idioms
Whichsentenceisidiomaticallycorrect?
A. Arisingof electricitypricesfuelledthe18percent
yearonyearjumpinthecompany'srevenuesto
3.6billionzlotys.
B. Arisein electricitypricesfuelledthe18percent
yearonyearjumpinthecompany'srevenuesto
3.6billionzlotys.
C. Araisein electricitypricesfuelledthe18percent
yearonyearjumpinthecompany'srevenuesto
3.6billionzlotys.
185
VERBAL
GMATClub contributingtoeachotherslearning
Idioms
Correctanswer:
A. Arisingof electricitypricesfuelledthe18percent
yearonyearjumpinthecompany'srevenuesto
3.6billionzlotys.
B. Arisein electricitypricesfuelledthe18percent
yearonyearjumpinthecompany'srevenuesto
3.6billionzlotys.
C. Araisein electricitypricesfuelledthe18percent
yearonyearjumpinthecompany'srevenuesto
3.6billionzlotys.
186
VERBAL
GMATClub contributingtoeachotherslearning
Idioms
Whichsentenceisidiomaticallycorrect?
A. Itseemslike thecompanyis slowlyrunningout
ofnewfeaturestoadd.
B. Thecompanyseemsasifitis slowlyrunningout
ofnewfeaturestoadd.
C. Itseemsasif thecompanyis slowlyrunningout
ofnewfeaturestoadd.
187
VERBAL
GMATClub contributingtoeachotherslearning
Idioms
Correctanswer:
A. Itseemslike thecompanyis slowlyrunningout
ofnewfeaturestoadd.
B. Thecompanyseemsasifitis slowlyrunningout
ofnewfeaturestoadd.
C. Itseemsasif thecompanyis slowlyrunningout
ofnewfeaturestoadd.
188
VERBAL
GMAT Club 2011 114
GMATClub contributingtoeachotherslearning
Idioms
Whichsentenceisidiomaticallycorrect?
A. Sweetpotatoesarenativein CentralAmericaand
Peru.
B. Sweetpotatoesarenativefrom CentralAmerica
andPeru.
C. Sweetpotatoesarenativeto CentralAmericaand
Peru.
189
VERBAL
GMATClub contributingtoeachotherslearning
Idioms
Correctanswer:
A. Sweetpotatoesarenativein CentralAmericaand
Peru.
B. Sweetpotatoesarenativefrom CentralAmerica
andPeru.
C. Sweetpotatoesarenativeto CentralAmericaand
Peru.
190
VERBAL
GMATClub contributingtoeachotherslearning
Idioms
Whichsentenceisidiomaticallycorrect?
A. Everyproduct usesimilar recycledpapers,
cardboardandothermaterials,inadditionto
leadfreepaints,nontoxicgluesandinks.
B. Allproducts usesimilar recycledpapers,
cardboardandothermaterials,inadditionto
leadfreepaints,nontoxicgluesandinks.
C. Eachproduct usesimilar recycledpapers,
cardboardandothermaterials,inadditionto
leadfreepaints,nontoxicgluesandinks.
191
VERBAL
GMATClub contributingtoeachotherslearning
Idioms
Correctanswer:
A. Everyproduct usesimilar recycledpapers,
cardboardandothermaterials,inadditionto
leadfreepaints,nontoxicgluesandinks.
B. Allproducts usesimilar recycledpapers,
cardboardandothermaterials,inadditionto
leadfreepaints,nontoxicgluesandinks.
C. Eachproduct usesimilar recycledpapers,
cardboardandothermaterials,inadditionto
leadfreepaints,nontoxicgluesandinks.
192
VERBAL
GMAT Club 2011 115
GMATClub contributingtoeachotherslearning
Idioms
Whichsentenceisidiomaticallycorrect?
A. Theleadinfrom'DancingWiththeStars'willbe
high,andsotoo theexpectations.
B. Theleadinfrom'DancingWiththeStars'willbe
high,andalso theexpectations.
C. Theleadinfrom'DancingWiththeStars'willbe
high,andsotoowillbe theexpectations.
193
VERBAL
GMATClub contributingtoeachotherslearning
Idioms
Correctanswer:
A. Theleadinfrom'DancingWiththeStars'willbe
high,andsotoo theexpectations.
B. Theleadinfrom'DancingWiththeStars'willbe
high,andalso theexpectations.
C. Theleadinfrom'DancingWiththeStars'willbe
high,andsotoowillbe theexpectations.
194
VERBAL
GMATClub contributingtoeachotherslearning
Idioms
Whichsentenceisidiomaticallycorrect?
A. Theaveragepricehasrisenatleast1centper
gallonperdayforeightconsecutivedaysto
$2.952,10centsasmuchas aweekago.
B. Theaveragepricehasrisenatleast1centper
gallonperdayforeightconsecutivedaysto
$2.952,10centsmorethan thatofaweekago.
C. Theaveragepricehasrisenatleast1centper
gallonperdayforeightconsecutivedaysto
$2.952,10centsmorefrom aweekago.
195
VERBAL
GMATClub contributingtoeachotherslearning
Idioms
Correctanswer:
A. Theaveragepricehasrisenatleast1centper
gallonperdayforeightconsecutivedaysto
$2.952,10centsasmuchas aweekago.
B. Theaveragepricehasrisenatleast1centper
gallonperdayforeightconsecutivedaysto
$2.952,10centsmorethan thatofaweekago.
C. Theaveragepricehasrisenatleast1centper
gallonperdayforeightconsecutivedaysto
$2.952,10centsmorefrom aweekago.
196
VERBAL
GMAT Club 2011 116
GMATClub contributingtoeachotherslearning
Idioms
Whichsentenceisidiomaticallycorrect?
A. SponsorsincreasinglyusetheInternetlikea
directmediachanneltocustomers.
B. SponsorsincreasinglyusetheInternettobea
directmediachanneltocustomers.
C. SponsorsincreasinglyusetheInternetasa direct
mediachanneltocustomers.
197
VERBAL
GMATClub contributingtoeachotherslearning
Idioms
Correctanswer:
A. SponsorsincreasinglyusetheInternetlikea
directmediachanneltocustomers.
B. SponsorsincreasinglyusetheInternettobea
directmediachanneltocustomers.
C. SponsorsincreasinglyusetheInternetasa direct
mediachanneltocustomers.
198
VERBAL
GMATClub contributingtoeachotherslearning
Idioms
Whichsentenceisidiomaticallycorrect?
A. Hazhadawayofspeaking thatwaslesslike
givinginformationandmorelikewarningsand
omens.
B. Hazhadawayforspeaking thatwaslesslike
givinginformationandmorelikewarningsand
omens.
C. Hazhadawaytospeak thatwaslesslikegiving
informationandmorelikewarningsandomens.
199
VERBAL
GMATClub contributingtoeachotherslearning
Idioms
Correctanswer:
A. Hazhadawayofspeaking thatwaslesslike
givinginformationandmorelikewarningsand
omens.
B. Hazhadawayforspeaking thatwaslesslike
givinginformationandmorelikewarningsand
omens.
C. Hazhadawaytospeak thatwaslesslikegiving
informationandmorelikewarningsandomens.
200
VERBAL
GMAT Club 2011 117
GMAT Club contributing to each others learning
Recommended Verbal GMAT Books
Ultimate Verbal Sets
MGMAT Guides + Verbal OG 2
nd
ed
Veritas Prep Guides + Verbal OG 2
nd
ed
All-in-one Verbal Books
Kaplan Verbal Workbook
PowerScore Verbal Bible
GMAT book reviews: http://gmatclub.com/books

GMAT Club 2011 118
GMAT Club contributing to each others learning
Best GMAT Courses
Manhattan GMAT Course - $100 discount
Classroom courses in many cities and online
http://gmatclub.com/mgmat
Princeton Review GMAT Club $150 discount
Classroom and online courses from $499
http://gmatclub.com/princeton
Veritas Prep 10% discount all products!
Save up to $650 on Veritas Prep GMAT courses as well
as Admissions Consulting Packages
http://gmatclub.com/veritas
GMAT Club 2011 119
GMAT Club contributing to each others learning
About GMAT Flashcards
Thank you for downloading GMAT Clubs GMAT
Flashcards. These flashcards have been put together
by a number of GMAT Club members ( Walker,
Bunuel, Shrouded1, Dzyubam, Whiplash2411, and
others)
Please visit http://gmatclub.com/flashcards
for the latest version of the GMAT Flashcards
If you have any suggestions how to improve these
flashcards, please visit the same page to leave your
feedback: http://gmatclub.com/flashcards
GMAT Club 2011 120

Das könnte Ihnen auch gefallen